Você está na página 1de 297
By DIT GILLESANIA Civil . eure Reference l\wou (Oa, | as or Tete i Coaty See 81] TUT tea MUI LAT) i} and Transportation fe ye) ene nt] Bo Serenity] Structural Engineering and Construction Civil Engineering Reference For Licensure Examinations Volume 2 Copyright © 2006 by Diego Inocencio Tapang Gillesania All sights reserved. No part book may be reproduced, stored in a retrieval system, or in any form or by any mans, prior permission of th ISBN 971-8614-34-6 Cover design by the author. Civil Engineering I Reference Vol. 2, Table of Contents CE Board May 2000... ‘Mathematics, Surveying, gene Engineering, 1 solution i ms Hydraulics and Geotechnical Engineering - Z 2 canes, pie 16 ‘Structural. ee ‘and | ‘Construction. abet 22 Solution. is 29 CE Board November 2000... «34 to 62 a Surveying, and ecreeteaies Pacing 4 3 pcos Gegecnkal ehnoaing : “4 Sain 7 crt aaiind st Camas ef oe 7 CE Board May 2001... 03 to 97 ar israel isn sig ion pose 8 Solution, amar, ‘Hycraulics and Geotechnical Engineering 76 Soltin % Gea ae el Cec 7 Salton 2 CE Board November 2001... .. 98 to 137 pears eee péTeansporeton ngnecing svcturalEnginering and Constaction Solution ” Table of Comtemk cn Board May 2002. 139 to 187 tics, Surveying, and Transpo 9 bution... 13 Hydraulics and Geotechnical Engineering. 154 Solution. : 160 Seructural Engineering and Construction. : 169 Solution 7 CE Board. November 2002 .- 188 to 237 Mathematics, Surveying, and Transportation Engineering, 188 Solution. 192 Hydraulics and Geotechnical Engineering... 202 Solution. 208 ‘Structural Engineering and Construction, 218 Solution 229 CE Board May 2003.... 239 to 291 ‘Mathematics, Surveying, and Transportation Engineering, 29 Solution 242 251 258 269 5 280 ..292 to 339 292 e 295 Hydraulics and Geotechnical Engineering. 304 Solution ~ 3 ‘Structural Engineering and Construction 322 Solution, 331 Civil Engineering m Reference Vol. 2 341 to 388 Surveying and Transporea x1 345, 355 z 363 Structural Engineering and Construction 3A Solution S 381 CE Board November 2004 389 to 436 Mathematics, Surveying, and Transpor 389 Solution 394 Hydraulics and Geotechnical Engineering “405 Solution, 4. Structural Engineering and Construction : 2 20 Solution 426 CE Board May 2006... 437 to 480 Mathematics, Surveying, and Transportation Engineering 437 Solution, 442 Hydraulies and Georechnical Engineering z 453 Solution 458 Structural Engineering and Construction 455 Solution Sia 474 CE Board November 2005 ... 481 to 526 Mathemaries, Surveying, and Transportation Enginecting 431 Soliton ‘ ABS Hydraulics and Geotechnical Engineering 493 Solution, 501 Structural Engineering and Construction 510 SOFUOD on ea 57 Problem Index... Table of Comtemts Referense VoL 2" 1 CIVIL ENGINEER Licensure Examination ‘Thursday, May 11, 2000 INSTRUCTION: Select the correct answer for each of the following questions. of your choice on the answer sheet provided. STRICTLY NO ERASURES ALLOWED, Use pencil no. 2 only MULTIPLE CHOICE 1. How tong will it take for money to quadruple itself if invested at 20% compounded quarterly? A. 107 years ©, 95 years B. 63 years D. 7:1 years“ 2 Find the aes in sg. m. of spherical triangle Of whove angles ae 12%, 8, and 73°. The radius of the sphere is sare 3 18633, © as585 D14802 © 6am B Oem D. idem 5. Five thousand pesos is deposited at the end of each year for 15 years ito an ‘account earning 7.5% compounded continuously. Find the amount after 15, oA PIS, 5413, C. P148.3659 B PIb%7542 D, Pui2.a541 6, There are four geometric means between 3 and 729. Find the fourth term. ‘A St 243 B27 Dd. 7 Loge 975-2, Find x 331 csi7 B 412 D. 287 2 & Evaluate fercoon A 15471 © c 08 B 19085 D. 20412 9. A projectile ged at an ange of 2" with the horizontal at the top ofa 0-m high building. “The muzzle weocty i 300 m/s. Whats the total ime of” oe Z 3268 © 22 B75 Di. 1855 . Mathematics, 6 and 2 May 2000 Sanson erleeeriy iO Two verses ofa wangio are (Gol) ACTS), Find the onde of he hog verter rach tut Be oammotd ofthe Giang wl ie at the origin x cn Be ba 11. A machine costing P80 000 hos a life expectancy of 12 yeas with a salvage alae OF 08 of Sk rot cost Whot iste book value oer ve yous acing Tpadectrng balercn atid? a Pisses . Pi96402 B Pise768 D,ratee 12, Two sides ofa tiangle measure 18cm and cm. The third side may be: 23 10 Bb bat 13, Atwhat value of x will the slope ofthe curve = 9r-y =0be 18? A eo oe ieee det ese f Fk. The usta ofa nega tinagelar py aden as eqn angie frie {aeetyeh eaweriairey fh arena | iaaeheaiey cian yet ete Soe ctae wae es om D3m 15. A cyinder of radius 6 m has its axis along the Kasi, A second cylinder of ee tees tthe volume onthe at coment etic ce © tm B tisem 3. 2as me 46, Rationale the fellowing = a-set sale - ab fee ” 1-8 17 A ightouse is 2m a stig shore. seh Ht the pou fecties toa ear moving along the shore. When the car 1 ko fom the Point nearest the lightiouse the searchlight rotates O25 rev/hour, Find Peipener tocar icon arss3 c 20 B 250 5.337 18, The total surface area ofa closed cylindrical tank is 15994 square meter. tf Hisvolina is oe maxtor, whats is hepitincmaers? Cem Dee 8 ertigabout the X-axis, ofthe area bounded by “4, and the Xeaxis. ©. 1017 D. 1219, Civil Engineering Reference Vol. 2 3 20, From pint A ona simple curve the perpendicular dotance to te lngent, z at poiat Qa 61 m. The tangent passes thvough the PC The distance froin teBC ie Sobm. Bind the eng of trecavetrom PCto Aina A m037 B snr The ground makes a uniform slope of 48% from STA 12 + 180 to ST 240. AUSTA 12+ 180 the center eight ofthe roadway i 12 mil anther sation, the center height is 25 meu. ind the length of cut in meters, ‘A. 30.85 e: sil B. ast ©. 4632 D, 50.28 A vertical parabolic sag curve has tangent grades of -"2% and +0.6%. Ifthe grade changes uniformly at 0.18% per 20 my find the length of the curve. ‘A. 200 mt C. 300m B. 150m, D. 50m The observed interior angles of a triangular piece of land ABC are as ‘A= 391437" (C= 48 15°05" B= 96° a0 09" The most probable value of angle B is nearest ‘A. 96° 50" 15' ‘C. 967 30" 06" B. 96° 30" 03” . 96° 30°12" ‘A closed traverse has the following data: Line Bearing. Distance (m) AB ae a 60.00 BC Sm 268 O siraweE 443 DE «8S 736 W 56.45 EAN 730 W 30.00 Find the bearing of line BC. A. S45" 52" C. seras'E B NWSI E D. Nis*s0'E ‘The perimeter of an ellipse is 28.448 units. If the major axis is 1D units, what is length ofthe minor axis? A oe B7 D6 Ina certain stretch of the north expressway 1200 vehicles passes per hour. If ‘the traffic density is 25 vehicles per kilometer, what is the space mean speed inkph? Ads C8 BSD D. 36 ‘The driver ofa car travelling at a certain speed suddenly sees an obstruction ahead and travelled a distance of 72 m during the perception-reaction time (of 25 seconds. Determine the car's speed of approach in kph. Ar ae C1087 B. 96.3 D143 A certain copier machine cost P150,000 with a trade-in value of P15,000 after ‘making 600,000 copies. Using the declining balance method, what is the bbock value when the machine had made 300,000 c A. P6811 c B. Pos.896, ° Mathematics, Surveying, and 4 May 2000 ‘Transportation Engineering 5. Determine the equation of the it crete) ov} feed is mm 20, Fit dhe Sette of aright circular cylinder whee Intra area is 25.918 a C. 20.53 m> B. isan fear E=A+ B+ C-180° = 123° + 84° + 73°- 180° = 100" id ie a = 30)*(100") _ x Area = SOG) — 15708 sq.m. ee te ie oP Avznrk 100.53 = x4) Lj L= Bem waned = PIS3541.3, log 9 975 =x x= log 975 / log 8= 3.31 Mathematics, Surveying, and 6 May 2000 "Transportation Engineering a0 ‘Thus, among the choices, x may be 13 Mathematics, Surveying, and 8 May 2000 ‘Transportation Engineering a, fixe Va-Jab a fab dp/at= 0.25 rev/hour i0/dt = 0.5n red/hour tan 0 =x/2=05x When x= 1; 0= 26565" a sect 26565" (05 2) = 05 gros & ~ssouayre maximum when the For a given total surface area, the volume is a hen diameter (D) equal the height (1) , Ags $D1x2+nDH=2 GHP + (HFN = 15394 B-572m L= fr (-2y'? + 4)dy Ayn, 2 he [orevene- be c ty] 12.9 Civil Engineering ‘ Reference Vol. 2 ‘20 In right triangle ABO: BE = 260" + (R- 64)? Re = 67600 RE 12aR + 4096 R= 560.125 m. Length of curve from PC to A: Te 1RO/ 180° 1. m(360125)(27 657°) / 180° = 27037 m From th ran << Janstn From the grade diagram shown: L Ey 12206 018 = 200:m Mathematics, Surveying, and 10 May 2000 “transporstion Engineering ¥ angles =35° 14’ 37” + 96° 30" 09" +48° 15" 05" E angles = 179.9975" = 179° 59°51” ‘Since the sum af the interior angles ofa triangle is 180°, the total error Eis, E= 180° - 179° 59 51” = 0° 0 09" ‘Most probable value of B = 96° 30° 09" + (0° 0 09")/3. Most probable value of A = 96° 30 12” ay 28 n/m = 28S m/s (1 han/ 1000 (3600s v=103.68 kph rh 28 Solving forthe closing line AC Tne Bearng Distance _atitude Departure By cD S17 20E 4483 42.7942 13.3562, sons = P6S/2545 De Smrsew © S645 187505 53249 a» TA N73 W 5000133619 48.1815, Beis, ‘Sum 48.1828 ——_—-88.0702 tae Latitude of AC = +48.1828 Equation of ditectrix = y= 4 ory +4=0 Departure of AC = +88:0702 Bearing of AC=N 61317" E ‘Length of AC= ((48.1828)' +(88.0702)° = 100.389 m O30 Jn triangle ABC: Ayn n= 7.068 “00 359 = 60! + 72.692 2(60)7269) cos f t=15m $= 97 8682" Ardarh =2n (15) k= 25: 28 eT IE eee egies Ly le a a V= Aa x h = 7.068(2.75) us Oue-a= 61.31 58292" ~ 15,4808 bat Oe) Ope B- 4a = 97.8683 ~ 15.4808 = 82.3875" Bearing of line BC= § 82° 25° E Hydroulles and Geotechnical Rngineering 12 May 2000 Seat Now Mh ee ee eco pipiens Al fl SETA HYDRAULICS & GEOTECHNICAL ENGINEERING i comet answer fr each of the follovring questions INSTRUCTIONS Sel 5m by soaing the box coresponding to he eter Spiyane answer or : onl SHEN hee provided Se nets ALLOWED. Use penilne 2ony MULTIPLE CHOICE 6 m at the side of a tank containing a quid VOSS es fat elevation 4 m reads 100 kPa. What is the: reads 60 KPA. Another gege ong gy ofthe ae 2 ee cnt ee wom ga an ag Te a ea hek oe a a. flay ommend nate ede PS from he con no Eon te eng eo mapa howe eens Bieron onic bahamas eas PI BL “c.uL/s ee liameter are 7 Bee and 2 of the mame Teng end dlamet connected in eee pee * paalle. If f= 2/3 fi the flow in pipe 2 is how 1 times gr Py 1487 ne Dre J. If the water 5. Wiel Ree over a sie and ints a hermntal cngl tigen pn ste B Sapcrecal Breet, rT iy ses sloping “ © AERP Sf ot, whats he hydraulic radius forth licen ‘1D, 0.86 m ti 1 nT ama ancnp tom tine toad 2m ih eee ny dam 4m ght TE yeni up tthe el ee atthe heal in kg/m? alecing Wye ght of maxon 08 2400 ke my C. 32,220 oa ‘1D. 39,780 B 34,750 Civil Engineering = Reference Vol. 2 8 ‘A cylindrical tank 2.m in diameter is divided into two equal chambers artition along its diameter, and provided with 20 cm x 20 cm orifice at the f the partition. Ata certain instant, the level in one chamber ‘will it take for the gular flume of mos efticient proportion has a base width of 3m, It ‘na slopes, has a roughness Coeficent ny and the discharge is 9 cu ‘m/s when flowing fall Ifthe same material were ueed for the mort tient apesidal secon, by how much Woud ti dsdarge be ‘A Ost m/s O71 m/s B. O61 m/s D, O8tm/s Thice reservaits A,B, and C are connected respectively with pipes 12, and 3 flning at a common junction Pwhose elevation is 3 m. Reservoir Ais at vation S33 and reservoir Bis at elevation 644 m. The properties ofeach pipe are as fellows: Ly = 1500 m, Ds = 600 mu, ft = O02; fa = 1000 m, Dr= 5330 mm, f= C0251) 900 my Dy = 500 mum, p= OB, A prossure gage at junction ® read 4950 KPa, What isthe flow tk pipe 3 i ¥/s ‘A. O91 cu Bass B. 076 ‘A rectangular footing is to support two square columns each 12" x12” and spaced 1 fect on centers: One column carnes a toed of 40 Kips and the ther caries load of SO kips, The footing is2f thick and its length should exerd 25 eye th cnt ofthe column crying the 40 od The ‘of footing is 5 flow rurface Resta specie gravity of orevete ant soil above the footing to be 24 and 18, respectively, Determine the length ofthe footing ithe allowable aol bonring capacity 20 pt c m8 5 isk Dist A srllay, 95.5 fect ong, whose crest i elevation 4.0m releases water from aeseryou whose surface area s 4.0 hectares, inal reservoir elevation isHL8 im. How Tong wil take forthe water level todrop I meter? 30m © 30min Donia The spillway with crest elevation of 5D m releases flood runoff from a 5.) hectates in area If the reservor level stops from elevation 32 ‘15.521 in 1250 seconde, find the length ofthe spillway in mets. Use the Francis Formula. Assume corotant reservoir surface arc, C36 D328 The ratio ofthe volume of water to the Viume of vod, "A mlstuce content degree of stration Bi vold ratio D. porosity Hyerautics ane ceil 14 Mas 000) Geotechnical Engineering 14 May 2000 Reference Vol.2 1s TE The ete Sage rer Tera Tay EE a Content Conga, apparent by by ply meron oa ieee ce to in emoval of must, earthy, ihy, and muddy tastes and ole eae (3) en ble gas into to atoms of oxygen. eae i ; 2 Te dy secant, u ; oieraene Meee rete pemeet yee can Benaet oe aererie tas cum/see, Water 8 178 Pate boc of tes et Fa 20 1651 {fe he head 80 om Consider the cechy operat ed se we once hoc ri an WP eo smn cae we i millimeters) om weegom Se retary bine i t ei To ug an res tte er el nek ude es Fonzonally aay rom the vena contracts, What ipdavalue ct eed om 16. Water flow in a 10 feet wide rectang rate of 64 cubic feet B. 054m D. 063m tea What te etical dep END ‘a moisture content of 30% and degree of saturation of se apecific gravity of 261. Determine the dry unit t C934 B wie D, 1025 1s. APaaple of moist day i found to have moisture content of 100% 5! 4 SemtPeg saturation of 5%, The specie gravity of the solide is 276 ‘Determine the voids ratio of this sol A133 C1028 5, 98 D, 1299 wp. 1 of moist sand taken from the field was found to have A Stant of 14% and a porosity of 38%. In a laboratory tions, it was found that at its densest state, Te Jousest state its void ratio is 40%. Determine the ‘A. 0527 C06 2B, 0854 D. 0A72, 20, Hild density tt on « compacted fill of sandy clay gives the following results: "Weight of moist soil from the hole = 1038 grams Overvdried weight of the soil = 914 grams Volume of test hole = 0.01 Laboratory moisture density test on this sil indicated @ mazimaty ¢O) Yaborston of pet at an optimum moisture content of 114, Which of te fellowing mast nearly gives the percent compaction of A. 963% ©. 887% $B. 972% D, 993% Hydraulics and 16 May 2000 Geotechnical Engineering Solutions to May 2000 Examination (6-4); y=10KN/ms 9p. gr = 10 / 981 ~ 1.02 Let = sp. 1 ofthe block weer e | Vo= SVs ‘AGH 20) = {At = H-20= SH > (1) a | Init iq te Vo: 0: 08 A(H~15) = S(AH) (o=09 OsH-12=SH >@) [sH= se) H-2=08H-12; e H= 40m Upstream from the leak: ve a pet on AoE p= QESAOO SOG? 4254, 09-025 m/s Qasim Qu~ Qo 0.216 ~ 0.205 = 0.011 m*/s (Qasen = TL liters per second Civil Engineeriny Reference Vol. 2 17 if = i 0.0826 L404? D> = 9.0826f21202" where Ly f= 2/f Oe Aonn ganar Pw 24 14142= 4,823 m R=3/4828=0521m Wy = 2400(14(12)(28)(1)] 4nk—nm coe OM = 1,543,500 kgm Ryx = RM-OM 576,000 = 5,990,400 - 1,543,500; F = 772m e=B/2- =028m Raees ener a (= al 7 ge 60.28" 16 16 = 39,780 kg/m? Prot 92.220 g/t Myarauites and Civil Engineering May 2000 Geotechnical Engineering Reference Vol. 2 & ao a) fyi 624m .0326(0.02)650010," «og EY ee os 2)" 1399 m/e CAs f2g = 0.602) 28 cA. fg = 01083 = 2n6-~ 29825(0025)(1000)0,? (045) Qo" 0488 m/s (x/2(n/2) _2_ (5 _ Jp) Q1=Q:+ 0: x pe LDN) 1 = 2+ Os \ n/2+n/2 01068 99 - 0488 = 0.911 mis 7: £136, p= 4850 4a 12255950 the elevation of reseroir C: _ = sexo Sa EL. C=6706~35.54~ 635.06 m au Fo sauce Forthepremrein | ae Q=KARS For the rectangular section: Ana5) 45m Saka O79"! K=0.8075 For the trapezoidal section: A= 1G + 2)(1.732) A= 5,196 m2 R= d/2=0.866m Q= 080766.196)(0.86 Locate the resultant load by taking moment about the 40-kip load: Q=381m/s < 90x = 40(0) + 50(12); x= 6.67 feet From the figure: 1/2=25+667, L= 18.34 feet, Increase in flow = 0.81 m/s eb ON Another solution: ‘2000 — (62-4 x 2.4)(2) ~ (62.4 1.78)(5 - 2) ~ 1367-264 psf ‘otal Ioad = 40-+ 50 = 90 kips = 90,000 Ibs a Using treatin, Qn 90,000 Gwe. = 1.2714; Ory = 2814218 ia z : LxW=18345W= 6582 W=3.59 feet Hydraulics and Geotechnical Eaginecring sae et 1341089) | Vos co = 1488 seconds = 24.8 min. 2(50,000) 1 |, L=5.64m mer [Wes Wy = MCW, W,=MCW, + Wis Wi= 1250= w TMC W, Wye 946 ce ~ GeMoy W grams) = 64/10 = 6.4 ft/s per foot width 2 z a= f= 1SA 22.0835 gg V322 GMC=Se; 261(03)=045e; e= 1.78 ie 261 2 2 = 9.81) = 9.34 kN/a Yeo" Tee" TO GMc=Se 276(4)=085¢; e= 1299 Civil Engineering 2 Reference Vol. 2 mis a. 38-9613 =a 1-038 p= —fmmat_ 085-0613 (085-040 =0527 Bi Ne ppt IPaCHON = “Maximarmdry density sim ry dara Ory = 0.0169 fe? x (124)(2.54%) = 478.55 cc Yo 914 / 47855= 1.91 gram/ce = 119.18 pef 11948 Percent compaction = 2248 sox = 99.3% 120 5 First we neglect the velocity of approach to get the approtimate velocity of approach Oe 1s4Lin? 053= 1.841 (03% 1= 1753 m Bigselam hata OTA 1.753(1.2) Mem ln ee ton BE = PE = 00001860 = (0.003186)9] L727 m2 For the water jt: y=xtno-—£2 o-0 20,2 cost 0° 981(25) (25-25 tan 0° 509 3.502 m/s 03° 0 Y = 3502; H= 0,625 m a= J2gHt 22 May 2000 sen Couerracion ey ae LT Sato CIVIL ENGINEER Licensure Examination (08:00 a.m. - 01:00 pn SETA INSTRUCTION: Select the comect answ Niark only one answer for each item by of your choice on the answer sheet provided. STRICTLY NO ERASURES ALLOWED. Use pent no. only. wr each of the following questions. ihe box corresponding to the letter MULTIPLE CHOICE 1. Which of the following most nearly gives the maximum shear stress in the rivets in MegaPascals for equivalent vertical force alone acting on the centroid. c 10 D. 70 nls forthe equivalent moment alone C70 B74 D. 760 4. Which of the following most nea gives the maximum shear sre in the vets in MegePascals ‘A170 c 10 B. 160 D.130 situaon 2 Te system shown n igre MBS consis of igh bars 4, AC. BD, & CD hinged at points A,B, € and D. When P is z2r0, points Band C ie AIBC & lesan aight i) andthe terion on AB ven arrangement inthe m the tension on member AB in N. A860, C. 980) B. 1130 DB. 1020 5, Which of the following most nearly gives the value of P in N. A, 840 a) B. 1020 D. 1540 6. Which of the following most nearly gives the spring constant K for system ABCD in N/m. ‘A. 3450 c. 2040 . 1870 D. 2560 vo a Civil Engineering 23 Reference Vol. 2 Situation 3 ~ The three concurrent forces shown in the figure are in equilibrium, 7. Which of the following most neatly gives the value cf angle 6 AS C38" B. 42° D. 46" 8, Which of the following most nearly gives the value of angle fi rae 5 eos le B si" D. 76° 9. Which ofthe following most neuly gives the vertica component of the 778- EN force. A768 KN © B2kN B. 702KN D, 653KN Situation 4 - A prismatic beam 8 m long is fixed at the left end and simply jorted af the right end. The beam carries @ uniformly distributed load of 400 KN/m throughout its lengeh 10, Which ofthe following most nearly gives the deflection atthe right end due ‘to.uniform load when the simple support is removed ‘A. 257600/21 C. 184500/& D. 204800/ 6: ingest ney gives the deflecton atthe right end due acting at the right end, wih the simple support ©. 457/381 D. 537/581 ollowing most nearly gives the reaction atthe simple support. Cc. 1300 kN a saa D. 1000 kN Situation 5 - For the structures shown in Figures AN-07, AN-08, & AN-08, identify whether the structure is unstable, statically determinate, statically indeterminate tothe frst degre, and statically indeterminate tothe second iegree. the fist dey D. statically indeterminate to the second degree 15. Theslructure shown in Figure AN‘O9 is: determinate y indeterminate to the first degree D. statically indeterminate to the second degree structural Snginecring 24 May 2000 ‘and Construction ee Situation 67 For the spiral column shown in Figure ROSE: jowing, ost nearly gives the reinforcement ratio Py in try a 150 milimeters. © oom DOO jes the ratio of the ca Jima dimension in the c. 065 B,0ss 18. Using the Interaction Diagram, which of Inapdinum downward los that te co Situation 7 - Section 5.11.6 of the NSCP states the following for the combined, stressed members with rectangular or flanged rectangular or fl jenent rectangles overhanging flange width used in design shal thickness. ‘51.64 Sections located less than a distance d from face of support may be designed for the same torsional moment , as that computed at a distance d omerete bean 300 mm wide, 600 mm, and 5 m long 20.75 Mi 19. Which of the following most nearly gives the torsional moment on the beam due to the overhanging slab in kN-m. 20. Which of the following most nearly gives the value of the sum Ex? y, due ta the component rectangles ofthe section, in ‘A. 0. 045 B01 D. 025 21. Which ofthe following most nearly gives the value ofthe limiting torque Ti, in KN-m, A 20 Gana BS D. 20 25 ia 205 of ee estimated con x minor pay item in ered by a Change Order provided th the Approving Author contact ‘The quantities and contiact prices fora projet are a fl Quant Unit Price (pesos) 100 200 60 500 ci} 0 Fi} Item 5 150 70 22. Which of the following most nearly gives the total contract price of the 2 arso00 . Pscoo i, oso D.ss0000 23. Which of thefllowing is not considered asa major pay item of the project. A. Item 4 C. Item 3. a D. Item 5 by a Change Order: fem 2 flexural stress for laterally braced compact sections is 0.5 F, The allowable deflection is) 1/360 of the span. Three compart sections are. being considered, as follows, with their respective properties Televant of this problem (length units are in meters Section Moment of Inertia (x) Beam depth (D) W25% 66 0014 0634 W30%74 00186 O72 W24x62 0011s Ooi (Note: The symbol x*y means raising the quantity or expression x to the ing most measly gives the minimum section modulus Tarte ests fun! ares wil not os coated . 0.00528 D. oaosis lowing most nealy gives the minimum moment of Inertia ache fos ath detictc wet emcee 26 May 2000 pret 27, Assuming that shear stiess is not cri considered is the mest economic section ‘A W30x74 BL W24x62 of the sections being wdequate for the given load: C. W26x 66 D. None of the sections are adequate Damm Constracion TEER para 7 Ta Wechanacal and elacial Fares n Die mn lowing gives the duration ofthe prj in weeks De 30. Which ofthe following gives the earllst start of activity m in weeks: A oar DST Bs Figure ME-02 78K 28 May 2000 Figure ME-08 Figure RC-12 = f x 2 Figure AN-O7 Figure AN-08 Figure AN-09 END 30 May 2000 1367 + 400 2T cos 30° es T=10202N Atjoint 8 P= 2T sin 0= 2(1020.2) sin 30° P=10202N Spring constant, kas: P= kago-x= Kaac (05) = 1020.2 Kee 2040.4 Nj D Situation (7 109) For the forces to be in equilibrium they must for a closed polygon (as shown) By cosine law: 7782-= 62.22 + 64.5? -2(62.2)(645) cos § = 7578 22 _ 6S __78 Bysinelaw: ing siny ” ain7S7® b= 0+30°= 75.74", 0= 45,74" +74 B= 180" 45.75" + 5347°+ P= 180°, = 80.78° Vertical component of 77.8 KN force = 778 sin B= 76.8 KN 2 Situation 4 (10 1012) Deflection at B when the simple support is removed: _ wit 40008)¢ BEI ‘BEI _ 8 Deflection due to unit load at B PL _ 1" = PE AO srapser Nigauee pe B= 2800/EF son ery 3, 512/3ET Civil Engineering Reference Vol. 2 1B Situation 5 (13 015) Figure AN-07, (PIN-ROLLER) Reactions, R= 2+1=3 31 Equations, £ = 3 DETERMINATE Figure AN-08, (PIN-ROLLER) “Although ies are UNSTABLE ecru pinied support. Figure AN-09,(PIN-ROLLER) Reactions, R= 2+1=3 | Equations, E=3+2=5 (there are two internal hinges) | Since E> 2, the structure is UNSTABLE Oy ‘y= 975/500 = 0.75 e/h= 150/500 = 03 32 May 2000 cea oe 33 Reference Vol. 2 ——————— ‘From the interaction diagram GD Situation 9 25 t027) Pew 13 tsi 6.8928 = 8961 MPa | Man = BE. 500}? Lee ‘Ay a ek wacom 075 Py= (600)? 8961; P, = 2346 KN | Maas 625 kN-m Me Scale Situation 7 (19 to 21) ten 7 = 200 0m ae AO = 0:6(248); 5, ~ 3818426 mar? = 00038218 ee, S + 200%(600) «2 < mea ue ae 1= 1171875000 mm‘ = 0001172, Zxty= 0102 m? ‘The required section must have 1 0,001172 and 5,2 0.00362 ae | For W 25 x 66, 5, 0.00124 /(0.654/2) = 0.00379 (not adequate) Fr x = NTO | For 1730 x 74, S, = 0.00146 / (0.772/2) = 0,00878 (not adequate) ha OD y 0 nee For W/24 x 62, 5; = 0.00119 / (0.618/2) = 0.00385 (ad=quate) y= 1974686478 Nem = 19.75 kN } Thus, the adequate section is W 24x 62 Beene 20 | @ Situation 10 28 to 30) Oty Price ‘Total Percentage oo —[Jo00 | Topmoo | ass 200 —|— 600 —| “120,900 —| 35368 5_|- 500 | ~100 | sa.900 [i053 [250 | 400 —| “00900 | asm em | ~150—[—700—| “Tos;p00 | 22.7% ‘Total contract price of project = PA75,000 | "Total Project Cost 3" | 475,000 | | Minor pay item = Item 3 From the diagram shown, the critical path is: a-b-d-f- Item 4 (Major pay item) from 250 to 160 Decrease = 90/250 x 100% = 36% > 15% Duration of project = 65 weeks Earliest start of activity m=45 weeks ‘Change Order {tem 3 (Minor pay item) from 500 to 600: Increase = 100/500 x 100% = 20% < 25% No need of Change Order Mathematics, Surveying, and "Transportation Engineering Seat Now 34 November 2000 CIVIL ENGINEER Licensure Examination ; November 18, 2000 08:00 am.-01:00 p.m. SETA er for each ofthe allowing auctions hing the box conesponding to te ltr ‘of your choice on the answer sheet ided. aie STRICTLY NO ERASURES ALLOWED. Use pencil no.2 only. Mark onl oncanzwa foreach tm MULTIPLE CHOICE 1, Twenty-eight persons can do a job in 60 days. They all start complete, Five persons quitted the jab at the beginning of the 16% day. ‘They were feinforced with 10 persons at the beginning of the 45% day. How many days was the job delayed? ‘A. 58 days /C- 1S7 days B. Llddays D. 245 days 2. Point A is between points Band C. The distances of B and C from point A are 1000 m and 2000 m, respectively. Measured from point A, the angle of elevation of point B is 18° 30’, while that of point C is & 15. Find the difference in the elevations of B and C. Consider the effects of curvature ‘and refraction, Addn c 487m B. a26m, D, 521m 9, Find the area of the curve x? + y+ 6x -12y+9=0, ‘A, 1255q. units 925g. units B, 1132q. units D. 138 sq. units 4, Bind the distance between the foci ofthe curve 9r2+ 25y?~ 18x + 100y-116= 0. A? C8 Be Dia a 5. A right regular hexagonal prism is inscribed in a right circu : wwhote height is 20 cm. The difference between the circumference of the ‘Grce and the perimeter of the hexagon is 4 cm. Determine the volume of the prism, ‘A. 9756 co B 11a752ce solid if every plane section perpendit isosceles right triangle with one leg in the plane of the base. ‘A 21333 C. 18667 ce B. 20155 ce D. 20433 ce 7. Whatis the area bounded by the curves x and x= 4y? A. 60 6.065 B 7333, D. 5333, 35 ‘The slope of the curve at any point is given as 6x ~ 2 and the curve passes through jermine the equation of the curve. A G2 dy-y-62=0 C. Dat + Br-y- 62=0 B. x ty +62=0 D, 3x24 2x-y+62=0 Evaluate the iniegral of x dx / (x2 + 2) with limits from 0 to 1 A032 C. 0203, B. 0108 D. 0247 The area bounded by the curve y = sin x from x= 0 to x = x is revolved about the X is the volume generated? C, 3.450 cu. units D, 5.214 cu. units Jindrical tank having a-volume of 71.57 1 is to be constructed. ‘surface area is to be a minimum, what is the recuired diameter of the its Adm C Sm B 55m D.45m A flagpole 3m ands at the top of. side of a pathw 1¢ opposite side va agp et the flagpole subtends the same angle as the 22 ‘width of the pathway? ‘A 447m B 321m C 628m D. 8am ling at 60 kph applies a brake and stopped at a distance of 30 m. nt of friction between the tires and the road is 05, What is the road? A. 43% B 43% C 28% D. 28% ‘A man made e yearend payment of 00,000 to an account earning 8% 10 years. How much is in the account after 20 years? ce sae : C. P3,307,452.36 ing a certain commodit and P15,0 per unit fo is 420 per million entering vehicles, whats the average daily afl (ADT)? 5 A. 589 C412 D527 36 Noveniber 2000 —"“tranportstion naglnsering 78. Froma point A ona simple curve, the perpendi at point Q is x igent passes through 250 and PCiis at station 20 + 150. Ifthe radius A. 624m © 789m, B. 2547 m D, 632m s drawn ata scale of 125000. An error of 0 lent to how many moters inack C. 005m D. som ects two tangent grades of +6.5% and is the elevation of point The geome! What ie the fi aS B36 2, Solve for Bin the given partial fraction: xt 15x3— 923? 125-14 _ 817, m4, joving verticall = 2 seconds? C. 2d fps D. 35.6 fps 25. Given that tan A = 4/5, whatis the value of 48!.A=CosA A. 0579 © 0654 B. 0752 D. 0925 26. Given a regular hexagonal with consecutive comers ABCDEF, Ifthe bearing. ‘of side AB is N 25° E, whatis the bearing of side FA? A NIsW ic BAS W 27. ‘The perimeter of a triangle is 58 cm and its area is 144 sq. cm. What is the radius of tho inscribed circle? A. 497 em. C. 552em B. 965m, D. 32cm Peter timate of cores 1s em mad 37 t rectangular coordinate of a point whose polar c 652.43) 3.56) xis is inclined 60° with the base. Ifthe axis is 20 cm long, what is the volume of the cone? ‘A. 1524 ce C. 1245 ce D. 1689 cc nagnitucle of the space vectar Si 4j + 9k? C 12544 D. 11.045, END F Mashemeatcs, Sorrring mo 38 November 2000 ransportation Raginssring Solutions to November 2000 Examination ai Required number of man-days to finish the job = 28(60) = 1680 man-days 28 persons worked for 15 days, 28 - § = 23 persons worked for 44 ~ 15 = 29 days, and 23 + 1033 persons worked for t days until completion, = 1680; 1= 17.97 da the job was completed = 15 + 29+ 17.97 = 61.97 days delayed ~ 1.97 days m2 0.087 m 0.268 m ADB: yi = 1000 tan 18.5° = 384.595 m ‘AEC: yp = 2000 tan 8 15' = 289.985 m + 2) (0.268 + 289.986) as Reduce 27+ y?-+ éx - 12y + 9 = Oto standard form: SD Gx +94 y2— dy +36=-9 +94 36= 36 aa 25(4) = 205 Distance between focl= 2c* 8 units 25) (14.125) sin 60° « 6 ‘Ay= 51836 cm? riper pe eee f V=518.36 x 20= 10367.13 em* Vm Bart thn Va E (Art 4am + As) Al=A2=0 (40)(40) ~ 800 cm? v= So sano) cee) = 5.333 sq.sinite pee. pee ane yo32- 2+ 8 (5,3): 3-36)-26)+C; C= -62 yohta bat st oas-y- e240 Mathematics, surveying, and 40 November 2000 ‘Transportation Engineering as In @)]= 0.2027 Present worth of F = Present worth of annuity for 10 years F__ r00,000(1+0 mo 4 Fixed cost + Vi A 450,000 + (45 + f N= 2368.42 say lin Vaan 2x3 = 4.935 cu. units Bez aw No. of Accident rate x No. of entering vehicles 20 20 4N, N= vehicles Thon ong “Ni N= 892691 vehick In triangle OAC: tan 20=5/2 300-3 ORE = cos 7.162 = 162 mu Fe Pert =1,000,000 enon) P= pa95a,032.42. fe Mathematics, Surveying, and 42 November 2000 ‘Transportation Engineering er Actual error=0.02 + 25,000 = 500 mm = 0'm aga Sz 400 T5006 26:36 a S=1 em Aa 15(126.316)(0.03) = 1.895 m EL, B= 123,256 - 1.895 EL B.= 121361 m InEq. (1): 184m) =64 a@?-34q+64=0; a= 32and2 Dx+E 2 Set x= 2: @- 152) - 32) - 120) -14 = 0+ BG)EYE)+0+0 0-308; B= 3 mu = 100 #16. Velocity, = 4 = 100 -32.2¢= 100 32.2(2)= 35.6 fysee a Civil Engineering 43 Reference Vol. 2 as tan A= 4/5, A= 38.66" 4sinA-cosA _ 4sin38.66°—cos38.66° ee eee bar BeosA+sinA — 3c0538.66"+ sin38.65" 32 interior angles of the lot 180°(6 - 2) = 720° w= 720"/ 6=120" 0=25" O+a+p=180° = 180" -25°- 190° = 35" Bearing of FA =N 35° W Arm rs 1d = 729), 1=4.97em For a point whose polar coordinate is (r,8), x= rcos 8, y=rsin 6 X= 7cos 38° =5.52 y= 7 sin 38° =4.31 Rectangular coordinate of the point = (5.52, 4.31) = 20sin 60° = 17.32em ® 30) =1469313 cm? yy f Paso = 12.085 Pdraullcs and 46 November 2000 eotechniea! Engineering 15. Twoppe | and Dhaving te Tollowing properties are connected in series ‘Pipe l: Length = 1 ore Ds sbbmm f= 0.025 the required pipe diame ‘A O74 mm, B. 578mm 16, This occurs whenever the normal dec stmospheric temperature or 3 both with height changes anomalously Over a elatvely short C. temperature inversion D. surface heating . fatal materials B. potable water D. sewage Figures eg f = o i Figure 03 Figure 02 END Reference Vol. 2 47 Dynamic force, F= 2% y= © age ae om J2aHf orvt= 29H Dymamifoce= (4) 29H Dain aie tt Samming-op pressure head from A to Bin meters of water 24 +038) + 0-2) 5 In(0.84) + B-2) i Solving forthe velocity ofthe jet at the summit (highest point, A) =D Di= va cos 0 24,5 10605 60"=5 m/s cfeuenene © (ata = om fant +n? = +0? =5m/s Since the flow is continuous: | , Gonos pe e figure shown: pm (0.05)? (10) = Aa (6); An = 0.003927 mx? BF, + B= 35,000 ms BR= Bry an | BF, = 21,565.44 Tbs | 21,565.44 + BF; = 35,000 r BF,= 13434561 = eet gplalay ssf olan Creer ty BE: yw. Vo = 624 ((12)(12)H]= 13 434.56, Die MOO.” cake ABOgeaea LOO earl N= 1495 ft From the figure shown: as Q* G= O.- 0057875 @) Fim prAi=120 $ (0.4)2= 15.08 kN .0825(0025)(500)0; __ 0.0826(0.035)(400)05? Stee uae 035° oa = 1194 m/s = 0757905 72 In Eq (1): 0.7579 Qs + Qs = 0.05787 Wonk , 360080, 24 (Qs = 0.03292 m/s x eee (Qs = 2844286 Lit/day = 2 ei a6 1199); R= 15.26 KN 15.08 = 0.5(9.81) Say (LIMO) Ry= 15.26 kN 36), 19-3 Bt Bae ag 3+5 06(01x0a)E RP +R? = (15.26) +(05.26) in 132m R=2156kN bs a ef 50 November 2000 22-23 aesxs0pn)[ 14 1800) | YF, 723 ce elevation = 42+ Hi = 42.921 m A=4(15) + 3G)(L5) x2 A=105m R=A/P=105/10708 fata a ees qe = o (Pat - JARED ae on avs = som Hy = 9:0826(0.025)(1500)0" , 0.0826(0.020)(1200}07 Tre y Q G. (0.85)° (0.65)? ana = £*G0982) 6 sn) Ho = 24.066 ea G=189+189¢ _Gte nae SHE, 1894189e46 "i s 2i5g= est) PEROT ZOO” 2406608 eee D=0674 m= 674mm 2-048 : Porosity, n= =f Porosity, n= 2422 931 = 31% 1+045 52 November 2000 Sua Cormerectes (CIVIL ENGINEER Licensure Examination Sunday, November 19, 2000 ‘STRUCTURAL ENGINEERING AND CONSTRUCTION SETA answer for each of te following questions ‘by sang the box coresponding to he letter Of your choie onthe anew rovided STRICTLY NO ERASURES ALLOWED, Use penitno. 2 only MULTIPLE CHOICE - A project has been bid out by the Department of Pubic Works and ert otimate (AA) is $00 milion pesos, The Mehow fi prices Nghe than th jowable government estimate (AGE) or the Approved Agency Estimate GAR) loc higher or lover fan 708 efthe AGE” The allowable fovemment emate ACE) defined a ane at ha sm fhe AE and average ofall responsive bids, For the purposes of determining en 203 ofthe AAE or lower than 3, Ps0.788,60.00 D, Paoas9200 2. Which ofthe alewing ost nearly gies the value of the approved jovernment estimate (AGE) in pesos, en ‘P482,262,700.00 C. P476,456,200.00 B. P01 455'90.00 , be 52,100.00 ee ae Civil Engineering 53 Reference Vol. 2 a cd 5. Which of the following most nearly gives the distbation lactor at B on member EC, in percent, Use the modified K. 439 C78 Bae si ig Which of the following most nealy gives the moment at B in KiloNewton- Meter. © mi) a3 es 80 D3 Situation 3 ~ For the frame shown in ANC15, the force F acting upward at C causes a horizontal reaction of 100 KN at C: lowing most nearly gives the valu: of the force F in KiloNewton’ A100 C135 B75 Dy 120 8. Which of the following most nearly gives the reaction at A in KiloNewton, A148 C. 156 3 ia bis 9. Which of the following most nearly gives the angle in degrees thatthe reaction at A makes wih the horizontal ns posits counterdcckwse) Cue Di oo lock resting on a smooth surfa in Figure ME-11, ‘The graph shown in the figure, 10. Which of the following most nearly gives the acceleration of the block ‘two seconds in meters per second per second, Aw CB B45 D. 32 11. Which of the following most nearly gives the velocity o* the block after five seconds in meters per second. An. 120 B. 102 D129 12, Which of the following most nearly gives the total distance travelled by the block in five seconds in meters shed horizontally by one versus te wal A. 385 420 B. 980 D. a3 angular concrete beam has a width of 300 mm and an of 500 mm. The beam ig simply supported over a span of carry a uniform dead load of 25 KN/m and a uniform ive load of 40 KN/m. Adsume f= 21 MPa and jy = 312 MPa. Compression reinforcement if necessary shell be placed ata depth 80 mim from the guemortconpremion cance PS : ich ofthe following most nearly gives the maximura tension steel area for singly reinforced condition ® es C. 3960 B20 D. 3780 54 Structural Engineering ‘and Construction November 2OOO Situation 6 - A ps v7. 20, jowing most nearly gives the required tension steel are B. 3750 Which of the following most nearly gives the required number of 25-mum tension bars AB co BT Dis beam is as shown in supported end causes a-unit rotation 1 most nearly gives the value of ‘applied at the joment M in ©. 700 D. 710 Which of the following most nearly gives the reaction at the simple support c.170 D155 lowing most nearly gives the moment at the fixed end in KiloNewton-Meter. ‘A. -300 . -350 B 330 Di. -380 7A project has beet ‘The approved a ssponsive bids are the Department of Public Works and. te (AE) is 500 million pesos. The Bidder C - PASA218557.98 Bidder D ~ P389,122/897-44 Bidder E ~ P284,758,426.54 ‘The implementing rules and regulations of P-D. 1554 states that no award of contract shall be made to a bidder whose bid price is higher than the allowable government estimate (AGE) or the Approved Agency Estimate (AAD), whichever =, of lower than 70% of the AGE. The allowable not be considered. No negotiation to the level of the AAE/AGE. ving most nearly gives the maximum bid price for the ©. 50,000,000 000, D. 650,000,000 Which of the following most nearly gives the minimum bid price for the . P300,000,000 . Paoo,000,000 55 ation 8A circular timber beam 250 millimetre in diameter is a simple mot 4m. The bounties a unlonaly stoned led ef BN] fading is own weigh The allowable teenes are 18 MPa for Sending td Ma for shear paniel fo grain Allowable deflerton is 170 of 8S span length. £ = 6000 MPa, Which of tho follwing most neatly gives the value of w 20 that the Allowable bending ste wil not be exteded lint Corwen the eeu fasion Gejucriscumhiong sooner AIG C2 Ba Dis 2. Which wing most nearly gives the value of w so that the hearing stress will not be exceeded. C150, a) D.190 2B. Which of the following most neasly gives the value of w so that the allowable deflection wilfnot be exceeded. aid C6 BS D2 Situation 9 - A simply supported steel beam 6 m long carries a uniform load of 92 KN/m and an axial compressive force of 320 KN. Tre properties of the Steel section is as follows: ‘Area, A= 14700 mm? Flange width, Flange thickness, y= 16 mun Overall depth, d= 390 mm Web thickness, y= 19 mm ‘According to Section 46.1 of the NSCP, for members subject to axial compression and bending, Li + Le es © O65, & {= computed aise, viel strength of tel = 248 MPa f ending stash rae bending atest = OC, role lowing ont neat ier he compte eal ess in he bem det ssa fore alone acing oh the bse, in epoca Az . cw i BF D.32 25. Which ofthe following most neal gives the computed bending stress in thebeam due to the unr lad los acting on the ben in Mego aa en Bee B75 27. Which ofthe following most nearly gives the value the interaction equation, A05 . fe C07 - Bod D. 06 i} if i 56 November 2000 Structural Engineering ‘and Construction Figures: 200 Figure AN-15 ot? Figure ME-34 END Civil Engineering 57 Reference Vol. 2 ee ———_—$— $$ 5 $— 5 << — $< Solutions to November 2000 Examination (E = P500,000,000.00, 120% of P500,000,000 = 600,000,000 60% oF P500,000,000 = 300,000,000 ‘Therefore, only bidders A, C, and D shall be considered as responsive bidders. Average of responsive bids: ‘Average = SS0:234A5158 + $54 218,557.98 + 989:122,897 4 3 Average = P464,525,302.47 AGE = 4AAE + Average of responsive ‘AGE = ¥6{500,000,000 + 464,525,302.47) 1 Situation 2 (410 6) Modified Kan = 26/4) = 1.5 Kec= 12/4=3 /6 = 2 nd Ken = 28/4) = 15 2000 saceonsraction | Referentetyon 3 59 @ Situation 4 (10 10 12) 58 Novem F200" asm a= aomie Acceleration tins degen Veloty-tie tram ‘The given graph is that of Fores (P) versus time, But since acceleration (@) = Force / Mass, the acceleration-time diagram can easily be constructed as shown above, The area under the represents the chang starts from rest, the jeration-time carve between any two points locity between the points. Since the block Ray= 100 KN Ey=0 Ray" F=120 KN Distance traveled: Se Art Art Ast AL 5 = '4(2)(80) +80(2) + (2/3)(2)(120 - 80)+ 1200) 8413.33 m B= 180° +6 = 230.19 ber of 25mm bars = 37455. _ Bey 0.85(21 312. 7.63 say Seructural Engineering ‘and Construction Civil Engineering 61 = 500; R = 166.67 kN (67) = 666.67 KN-m GR = 666.67 ~ 6(166.67) =-833.3 kN-m. ‘Maximum bid price ~ 120% of PS00,000,000 = P600,000,000 ‘Minimum bid price = 60% of P500,000,000 = 300,000,000 Therefore, only bidders A, C, and D shall be considered ae responsive bidders Average of responsive hide: ‘Average = £50.234,451,98 + 54,218,567.98 + 389,122,89744 3 Average = P464,525,902.47 AGE = Y{AAR + Average of responsive bids) AGE 000,000 + 464,525, 302.47) = Pag, 262,651.23 70% pf AGE = P337,583,855.86 Accor to the rules and regulation, no award of contract shall be made to a bidder whose bid price is higher than either the AAE or AGE, (P900,000,000) or winose bid is lower than 70% of AGE (P337,583,855.86) ‘Among the responsive bidders, the cant be made to bidder A. It ‘may etther be awarded to bidders C or D, and among the two, Bidder D is more desi ©) Situation 8 (22 to 24) Bendit ‘Equivalent square section: (b= d= x) $250); = 221.6 mm 5M 221 6221.6) (646,041,088 N-mm = 32,645 kN-m 18 = 32.646; w = 16.82 kN/m ‘Structural Engineering 62 November 2000 ‘and Construction w= 147,3KN/m Deflection: 4-277 MP | Bending sess: 2 1-08. aa) M= 144 Wm M _ 14axt0" 5, 192ix10 Civil Engineering 63 Reference Vol. 2 CIVIL ENGINEER Licensure Examination Friday, May 11, 2001 MATHEMATICS, SURVEYING, AND TRANSPORTATION INSTRUCTION: Sek ‘Mark only-one answer ENGG. SETA he corect answer for cach of the following questions each item by shading the box conesponding tothe eter of your choice on the answer sheet provided STRICTLY NO ERASURES ALLOWED. Use peni no, 2 only ‘MULTIPLE CHOICE na 4% upgrade suddenly applies a brake, Ifthe jen the ites and the pavement is 03, how fa will applying the brake? 74m D, 61m 2A truncated prism having a square base has a volume of 100 cubic meters ‘The height of the prism at each corner is respectively 7m, Zm, 10m, and 10m, What is the area of the base? © 13432m D, 125.21 m 8. Two sides of a parallelogram measure 68 cm and 83 cm and the shorter diagonal is 42 cm. Determine the smallest interior angle of the A 2Bar C, 4945° B 3027" D, 12.65° 4 Water flows at the rate of 16 m'/min in conical tank 12 m diameter ot and 24 m deep. How fast is the water surface rising when thewater is 12 m deep in the tanic ‘A. 0.231 m/min © 0828 m/min B. 0712 m/n D. 0.566 m/e 5. Whatis the p the graph y= sin 2? An Con B 32/2 D. «/2 6 An open-top cy iaving an area 43.82 square meter. Ifthe diameter is 2/3 the height, what is the height the tank? A32tm ©. 423m B. 243m D. 523m, 2. An equipment ha of 450,000 has a life expectancy of 10 years with a final salvage value of P80,000. Using the double declining balance method, what is its book value after 6 years? ‘A PI7. 964.80 CC. P184,300.00 Z D. P230,00.00 Mathematics, Surveying, and Civil ok 64 May 2001 “Transportation Engineering Eeieertng ie —— . Find the effective rate on nominal rate of 85% compounded continuously. | 90. A circle of radius 9 cm i circumscribed about a tiangle w ‘A. 988% © 857% square em. If one side of the triangle measure 18 cin, B 887% D. 10.12% the shortest side of the triangle 9, How far is the line 5: ~ 4y + 7= 0 from the point (2, 8)? A437 C. 564m A 475 Sar Behm B.234m B 984 21. Inahydrographic survey, astatf gage reading of 8.15 m was cbeerved atthe 10, A ban alls rom is house ‘depth of the sounding was 17.6. The aero merk of the salt Walk at the rate of 2 kph, gage 6 at elevation 148.2 m, Find the elevation ef the peint whese the 8:30 and walk at 3 kph, he will arive 6 minutes late time should he Sounding was made eee trive ne 13025 an ©. 19625:m © B54 o'dock 3 13875m Di a773m B. 843 oeoa 22. The sum of the fist n ferms ofa series is 3s" 6, Find the fith term of U1, Find the area bounded by the curve y = 4 sin x and the X-axis from x= 1/3 series. oi tore | 38 use ‘A Sequare un C. Bequare units Larry D142 5 ioequa DB. Saquare uss 2. Two sides of triangle measure 1 cn each. Wat isthe maxinum possible 12. Whichof the following is nota prime number | area ofthe triangle? % C7 A Tat emt Stem B17 bn Beem" ae 12, Tee ott tance rom PCto FT of espe curve 3 m. Whe angle Of MNEDA. Evaluate to aut of (4) / = 2)ar Dace cree 2 Jrtessecuon ofthe curves 20 whats the rads of the curve? ‘aT cy A iD3e in C i5898 Ba B, 12265n D. 11362 s thot its acceleration asa function of ts 14, Determine the equal payme worth factor of an anmaity of ‘and isin m/min’. lis velocity P1500 per year for 23 years its Velocity after 2 mines. to | C45 m/min B 1389 | D, 18in/mnin 15. Determine the volume ofa regular tetrahedron whose side is 3. 26. 2 ging 200 N in placed 25 m fom the center of a crcular et 53am Tolating plationm having a sus of 4m, Ifthe plovorm ip folated st 13 B. 2983 m3 D. 1.119 m3 | pm, what minimum coefficient of friction between the block and the 16. The area inthe second and third quadrants ofthe curve 22+ y!- 9 0 is Platform to prevent the block fem siding? revolved about the line x9" O. Find the volume generated. ‘A043 C0629 A Sb4k cubic units G aaa af o2t8 D. O86 By 11297 cubic uals Was enbicunts 27. A 45 cm 45 cx square plate ABCD of uniform thickness is supported by 17, The chords ofthe ellipse 4a? + 9y8= 144 having equal slopes of % is bisected tivee vertical rings The fet string is locate at B, the secane i locoted oS byte diameter. Whats the equation ofthe diameter? cm from A along side AD, and the thd sat the point ineay of side CD, A. 18x = 25y=0 © Mr+17y=0 What percent of the load is carried by the first and second supports? B, l6e+27y=0 A 40% ©, 50% 18, t point A and B. 60% D. 30% the stadia interval factor is $8.96 and the stadia co C. 7332m B Baym D. 60.28 m | 19. A compound curve has the following properties: erg Length of long chord from PC to PCC [a= 285.98 m ength of long chord from PCC to PT, Ia = 17823m Find the length of chord from PC to PT. ‘A, 402.15 m_ ©, 176.45 m B. 37654m D. 23476 m ft Mamematies, survering,2n¢ | evil Engi 66 May 2002 Transportation nginecrME | Weferenee Vol, 2° 67 28. A closed traverse has the fl ing data Line |__ Beating | Distance AB_[ Neo | 35.20 Bc sass [350m COS W 7895 me a eS Nees a 30086 EA 2 2 B. 49287 6 = $D8+nDH;D=2H/3 (QH/3)H; H= 4.23 m EVarthe BEGINNING of year [aFe= P2y49120 PRED rst a ea sy.eeom( 1-2) = arpa cas ER=e~1= 09-1 = 008872 = 8.872% 4 os eee er ie May 2002 ‘Transportation Enginesring = departure time 8=t- 805 e=t+6, I 1¢* arrival time - departure time Travel time = (14+ 6/60)-85~ 1-84 S=0t= 3-84) 5=3t-252 | 9@) 2-161 =3t- 25.2 t= 911= 9:06 o'clock xox -0s 60° ]= 6 square units umber is a number that has no other factor other than one (1) i the choices 77 is not a prime number beccuse itis A, and 77. Mathematics, Surveying, and 70 May 2002 “Transportation englaecring On equal pay’ 1, a+08)%-1 7 018 = 342.603, ms Volume = = 3.182 cubie meters 62 By the second proposition of Pappus: V=Ax2 A=%AnQ)?=45n R=3+a=3+ 40) Ra3eang+ Re 47732 V=Ax2nR V 45x x 2n(4.2732) = 379.58 cubic units 422+ y= 144 Differentiate with respect to x + 18y =O Sety'=% 8x+ 18y(3/4)=0 loxr+ 27 y=0 1g PC ype om 152° | 1, = 23598 m 72 May QOL ean hence one side of bea is = 321- >) A= 18)1) sin = 98 in 6 | Aa NE " #4 93c0s0=0 cos 0~0;0 = 9° ae A= 981090" 98 em? an Ekevation= 1482 + 8.15 - 17.6 Elevation = 138.75 m CHEM ip woryeze2-e m2 ms 29 125 v= +129 a [esma (2-1) + 62-14); 0= 31 mYmin atbye _ W+ld+x sn ABYC 2 AEE = 14+ 05 2 2 s-a=05x 5 -c=14-05x i 5—b= 05x Am [a+ 05xyOSx)(OEAN(14 05x) Ax Jose (196-0252) = 025096." -025:") ern ve} and Civil Engineeris 74 May 2002 “framperstion socinecrig Reference Vol 2 75 as : 20m | 1m | 214m 3500m | 385m | 3.23. 7895m_| 243m | 7eadm 5870m | _5798m | 946m sam = 200 (0502 (25) =12576N ~94.85- 2451+ 57.93+x=0 9. x=-075m EDep= 395.14 +323-75.12+9.46+y=0 ma y=27.29m In Eq. (1) AR, + 6R) Ris 03W= Re Rit Ro= O6W= 60% ‘Thus, 6% of the weightiis carried by the first and the second supports. a+ 0=4i BG= 31,8198 sin@=2/ BG;a=1423.em tan = 22.5/45; 8 = 26.555" Difference = 150,119.4 -109,705,7 = 40,4137 m? MP4 Brt+4y-61=0 dace (2+ ert P+ dy + 4)= 61 +1644 (er4)2+ (y+ 2)? B1= 9 radius =9 Area = 72 = 2(9)?= 254.47 eg, units 76 May 2001 Seer re oer es eye) ba Neto CIVIL ENGINEER Licensure Examination Friday, May 11, 2001 02:00 p.m. 06:00 pa sets lowing questions inding to ICTLY NO ERASURES ALLOWED. Use pencil no. 2 only ‘MULTIPLE CHOICE 1. The crest length the Cipolletti weir is five tim 82m%/s, its head, Find the lengt © 189m Bam ahead D. 0936 m of oop of water and 0,6 m of A. 1267 kPa B, 8:23 kPa ruin Figure 23 carries the 8: m/s. Beat m/s J PK long pip cornet to eserves a ss i Yara 2. Tha bercicst ip 11 10% of the majarheadllost, Deteenine the diacharl pein m/e | A 1012 C0534 | B. 0327 D. 0854 | 8 Aclay Samy lowing properties: Minimum void ratio= 0.42 Specific gravity of solids = 2.72 | drautics ang Civil Engineering Geotechnical Engineering Reference Vol. 2 7 natural state in pot 5 D. 98.34 B. 110.36, 9. A soil sample in its natural state has a wet density of 1551 pel and a ‘moisture content of 36%. After compaction, its maximum dry density is 1185 pef. Determine the percent compaction of the soil. G= 2.65, Biz Coa 5. s543% D, o876% 10, ‘The ratio ofthe volume of voids to the total volume ofthe sol is call A. pore moisture content B. Segree ofaturation D. voids ratio 11. A 100-mm pipe is connected toa 75-mm pipe, Ifthe velocity of flov in the [d-man pipes 2 m/s, whats te velocity o flow inthe 75-hon pipe? Anat ts ©! $56m/= 5 287 m/s D178 m/s 12 In tow net construction of detemining secpuge fw, the paths of equal nergy called ‘A How disgrams . flow lines B. equipotential lines D, seepage lines 13, A dam 40 m high has a spillway discharging at 3 m deep and a crest length of 10 m. If Cy=82, ind the discharge in cubic meters per second, A. 8530 C. 10000, B, 81.60 D. 9050 14, A fluid with kinematic viscosity of 15 x 107 m2/s flows though a 75mm velocity of 08 m/s. The flow is ‘ulent C. subcritical D. supercritical pe of 1:1 carries 225 ft/s is laid on a slope of 0.0002 wi in feo. 15, water at a n= 0.015, D. 522 feet l properties, the ratio of the total mass to the total volume of solids is led: . bulk density 16, D. dry unit v7, th roughness coet ine is 0.0025, determine the discharge c 578 B 1132 D. 1498 18. For the pipe shown in Figure 21, v1 = 0» = 1.2 m/s, Determine the totel head lest in meters A.337m © 498m D187 0. shown in Figure 28 is Id the gate in position C. 46737 Ihe 1D. 5088.5 Ibe nged at B. Determine the 78 May 2001 ‘The ren Which A. 26% B ity index ation due to grav 19.21 Ibs B 5843 Ibs from the top of the sheet secured to two anche may be assumed he maximum pa ‘of a gage inclines ‘same gage installed vertically. ‘A. 96% per meter. is the weight of a 12-slug boulder if 3076 ft/s2. the anchored bulkhead show piling and are les raked pressure ‘Caleulate the compressive A soil sample is placed on a mixing dish and 15 - 20 ml of distilled water is then stirred, kneaded, an« 3 mi) water is added and used to determine the: ‘with a spatula. An additional (1 rocedure is undertaken. This per meter and 0 a fluid in the bee ergo unr rest poets | ena eee pale . = 1360 34 4 0 4 90 5S oo 65 Figure SM-3 Civil Engineering Reference Vol. 2 i Figure 32 “END >onoee| 20 2d ne Hydrauiles and Geotechnical Engineering 80 May 2001 Solutions to May 2001 Examination For a Cipolletti weix, Q = 082 = 1.8596 H= 0379 m; L=1.99m m2 as 1 = 6.862 m/s 0 Fi ggix? 1)? cos* OF aa = 2d b= 08284 3.6 ~ 1.828 2 d= 0.936 m as p=Zyh= 981008) + 0.81 0.82\(0.6) =1267 kPa Civil Engineering Ps Reference Vol. 2 8 ae a= Heusen, 36 0 = 6mm v=17.08 m/s E m7 For a pipe o reservoirs, y= 15.636m (major head loss) Ys me ee ive nm , _035 oee gente i I-n 1-035 ne 2m +0595 (624) = 110.36 pet ae Dry density of soil ¥ Maximum dry density “1% = 26512651096) 69 Tre i 2.65 tay Ee) = Yay = Fogag 624) = 114.04 pe 045 114.04 1185 Percent compaction = 100% = 96.24% Hydraulics and 82 May 2002 Geotechnical Engineering By continuity condition: = Qu Arn $O1PQ)= § 0075) vy; v2 = 3.56.mys Q=CyLEn2=3.20 905 m/s D . 2807S) - 40000 > 4000 (Turbulent) se aoe Po b+ 2y2a b=7.29-2828d > Q) = 1828 = 697 1.828 #-729d+697-0 d= 1.59 m= 5.22 feet a b= 1698-0. 1347 = 1.5633 ft Zz (ince »1= v3) - Hydraulics and Civil Engineering Geatocanlest emeiacecine MMIMCRg evar 85 Liquid Limit Test Cone Penetration (mm) 3645 KN (2.8) = 364.27(2.4) = 230.2 kN 5) = 230.2 Pa Pe 65(1.2) = 78 kPa (OK) 1096. 20.99% THF, z 21.04% T= 3645 ~ 2502 = 1343 kN | mt j ue a re1ai x 75.96" , =e A= 0) P= 52% - 21.01% = 30.99% Fj sin Oy = Fa sin > F)=0989 m2 [eFi= 0] Ficos0:+ Frcos 2=T F, cos 75.96° + 0.989 F, cos 78.69° = 134.3 F,= 307.6 KN (compression) ug, Hydraulics and 86 May 2002 Geotechnical Engineering aw fctaliay petcalr, the horizontal ae vertical Thus the gage catches Sere 15% less then when it is vertical (as given), When 0 = 20°, A;= 0.94 4, ‘Thus, the catch is 98% = 94% Ae he vertical gage. Pe seeerias: 87 Seat No CIVIL ENGINEER Licensure Examination Thursday, May 10, 2001 08:00 asm. - 01:00 p.m. STRUCTURAL ENGINEERING AND CONSTRUCTION, SETA INSTRUCTION: Setect the comrect answer for each ofthe following questions ‘Mark only one aver foreach item by shading the box coresponding oth eter ff your choice on the answer sheet proved. STRICTLY NO ERASURES ALLOWED. Use peneil no, only MULTIPLE CHOICE n 1 ~ A contractor was awarded a four-year contract for trucking He wishes to purchase pick-up truck worth P1,000,200 each. The 100 kim per year. The estimated fuel consumption per year is F125,000. The dealer of the truck agrees to handle all the maintenance cost for four years at an additional cost of P200,000.. Mor worth 15% per year. Salvage value of the truck after four years is P3000 1. Which of the following gives the annual cost of the truck not including the maintenance cost. A. Pads 872.29 S Pangs4s.o4 5.75 2 be i Oe tia eick ererrme oe C. P62,875.45 D, P51 332.65 3. Which of the following gives the total annual cost of the truck. A P432982.12 C. Pass 238.82 B, P512.893.42 D. 7623/6723, ‘The beam shown in Figure AN-26 is 150 mm wide and 300 mm allowable bending stress on extreme fiber is 8 MPa. If x= 2m: lowing gives the maximum moment that the beam can jloNewton-meter, Br Wn fll gerartstar voi osha Tol 8 KiloNewton per meter. ‘9 Bu A125 Situation 3~ A square tied column is to carry an axial dead load of 500 KN and ‘an axial ive load of 750kN. Assume f,= 21 MPa and f, = 275MPa. Use 28- 1mm bars and reinforcement ratio of 2%. Seruetural Engineering ‘ 88 May 2001 ‘and Construction Ban eeecinn’ 89 17. Which of the following gives the Y-component of the force in KloNewton, Which of the following gives factored load Py KiloNewtons, ‘the column will carry, in RS ey ae c 1975 Basal Dia? & Which of the following gives the required column dimension in mm. 18, Which ofthe following gives the Z-component af the fore KloNewron, aan een B. 7428 ‘D. 8214 & 302 Ba 2 ae of the following gives required mumber of 20 mm bars Situation 7 ~ For the truss shown in ne? AN-13, the product of the cross- a oe sectoral rca and aodulus of elastic i 200,000 Neston llowing gives the axial force on member BC due to the given of 280 mm and an Her Reet ee Sarat a0 c2 eam. having @ wi farced for tension = 21 MPa, = 0.85 f. 20. Which ofthe following gives the axial force on member BC du oa verial llowing gives location of the neutral axis from the outermost Saeed nee: ean compression concrete et © 135 C5 BOs D.1s D275 21. Which of the following gives the vertical deflection at Cin mum. n fatements is true forthe given beam A535 Cass the code D335 Sot Pin ried teu ou SE se al ro sos ctrode with P, = 550 MPa, Allowable D, The steel and concrete yields simultaneously 12. Which of the following gives the moment capacity of the beam in KiloNewton-meter. ‘A 1 c. 138 roperies of L150 x90 12: Bie Bie Ree Zainal 3-9 mm Situation 5 ~ A certain equipment has a first cost of P900,000, life of 8 years, and. ea Geer ich ofthe following gives the value of a+b in millimeters. 18 ing gives the book value of the equipment after 5 years ee “s os more Harahan - ine eto, D351 i eis 23. Which of of the following gives the value of b in millimeters. - 7 14. Which of the ‘lowing ives the book value of the equipment after 5 years ean 5.288 ‘using the sum-of-the-year’s-digit method. x ich of the following yes the value in KiloNewton. "A P389,395 00 C: 726732900 Be eaban: Sores ccame eee B PH12345.00 D. 31666700 6 D365 15. Which of the following gives the book val using the double declining balance method. Situation 9 - For the beam loaded as shown in Figure AN-36, ‘A P525,982.00 aaa 2. of the following gives the resu loads in KiloNewten. ere Liao ‘Which of the following gives the resultant of ons in iloNewto D. 50 26. Which of the following give the lotion f the resultant oud from the eft the equipment after 5 years Situation 6 —'The magnitude of a force is 1000 KN and passes through the origin and point (¢=2, y=3, 2 = 4), 16, Whit the flowing gives the X-componen of the force in KilNewton, : A Lk eras 2 S38 B. 4535 D, 3242 27. Which of the following gives the reaction at the left support in KiloNewton. ito ae Bas D. 35 91 bottom The beam laterally supported over the ‘a uniformly distributed load of 860 KN/m inc ee aly cere arg strani 04 Fy Allowable dtl 28, Which of the following gives the section modulus ofthe section in mv’ A. 00076 C. 00054 D, 0.0058 Which of the following gives maximum deflection of the beam in mun. C159 D148 ¢ for the given section. an Figure AN-26 Figure AN-13 Figure $t-03 let I0KN sDH ISHN SN Figure ST-17 END“ Structural Engineering ‘and Construction 92 May 2001 Solutions to May 2001 Examination 2 Situation 1 (1103) OM + annual depreciation Annual interest on investment = 1,000,000(0:15) = P150,000 (OM = P125,000 (operation cost “gas”) (FC-SV)i _ (1,000,000 ~ 300,000)(0.15) Annual depreciation ~ (+015) 4 Annual depreciation = PI Annual cost= 150,000 + 125,000 + 140,185.79 = P415,185.75 Annual cost of maintenance 200,000(0.15)(1 + 0:15)¢ AO Ai a 7 PDAS Total annual cost = 415,185.75 + 70,052.07 = P&85,238.82 {D Situation 2 (4 10 6) w(t) hae ‘Shear cogram = 36 g=-l2KN; Thus, P=6+12=18kN © Situation 3(7t0 9) P,= 14 DL +17 LL= 1.A(500) + 1.7(750) = 1975 KN Pe= 08 $085. (Ag A 1975 x 1000 = 0.8(07)[0: 3526; Ay= Hi? = 153,385.2; = 391.6 mm A= 0.02 A» 0.02(153,385,2) = 3068 mm? (20)? N= 3068; N= 9,8 say 10 1 Situation 4 (10 to 12) Ass FORE x5 A,= 3078.8 mm? 1 A,= 27,709 mm? frauen =945MPa a= 27,709 By-o) 2 2) = 27709(520 — e) 140 c2* 27709 ¢ ~ 14,408,680 = 0 c= 2368 mm c= k= (520) = 236.8 k= 04554; j=1-i/3 = 08482 In terms of concrete, the resisting moment. M4 fej kb d= 19 49)(03482)(0, ‘M= 138184021 Nem In terms of steel: M= Acfej d= 9078,8(128)(0.8482)(520) = 173817324 N-mm M=17382KN-m Thus, the moment capacity is 138.18 kN-m From the results, the concrete yielded first and thus, it daes not comply with the code. 94 May 2001 DSiwations (151015) 5 = 87500(6) = PA37,500.00 FC Ds = 900,000 ~ 437,500 BV; = Pa62,500.00, =2a+n-2a+a-: Sum= 5 +n)= > (1+ 8)=96 =" 51218)-5+11-. psa5.055,99 = 00000 - 200000, 520-1 - p5s3.39 20 BVs= EC~Ds Bs = 900,000 - 583,435.33, BV,= P316,66667 Double declining balance method; (n= 8) “a todatee 25 Vat begining ft yn Yo Teampy | VameEND of am F Tan 2 (. 7506250 |: 1 = \: | =: o z ‘Thus, the book value after 5 years is P284,765.625 ‘Note: Since the salvage value is P200,000, the minimum book value must bbe 200,000 and thus, the depreciation charge after 6 years must not exceed PI3,574.215 Civil Engineering Reference Vol. 2 98 Using the formal Sie Sees Te tvesa.0(1-2) 3) vy asst £2 sation 661018) Fe Sod Rough L= Yee —, 500N (compression) 600-+ 500(8/5) 900 N (tension) Sau 400.N (tension) Stress due to unit load: Atjoint C BAr=9] /9)=1 1.25 (compression) ‘w= 0} Une= 1253/5) = 0.75 (tension) Structural Engineering 96 May 2001 ‘and Construction o Usa=1 sion) ‘Wertical deflection at C: = Taga [Sun Uae ban + Sac Une Lac + Suc Une bac 3 ph ggg [Sen Ur ban + Sac Ue Lac Sec Use Lac] 2 (1) (4000) + 900(0.75)(3000) + -500) (1.25) (5000 23 = Sapa [80004000 + 9001075) 2000)+ (-500)¢-1.25)5000) 28°65 mm 1 Situation 8 (22 t0 28) Value of P: ain Pern A= (06) A 3 ‘| P= [0.6(248)]0751) F = 4095488 f P= 409.35 KN : sos tata Value ofa + 3 Pen7ant oe eee, 707 (10) L [0.3(550)] = 409,350 051516 + b= 351; = 251.7 mm © Situation 9 (25 to 27) 20K JOKN 30KN ISK SKN Resultant of loads: F=2-10+30+15-5 F=50kN Location of resultant Lona: 50(x) = 20(1.5) - 10/3) +5045) +15(6)-57.5) s=a75m V[smlism!ism!ism!ism Reaction atthe left support: [EMs= 0) R75) = 20(6) - 10(45) + 903) + 15(1.5}; Ra = 25 KN 5 = 0.007595 m? Maximum deflection: p= Salt 91360 SsiET ~ 385(200, 87137 mn llowable deflection = L/360= 16,7 mim V=Reaction= 110801000) (C50)(18) = 1080kN 360(6) 2 fn = 107.14 MPa > Fy (not adequate for shearing) ble bending stress, F, = 0.66(248) = 163.68 MPa ‘Actual bending stress: {f= 2183 MPa > (not adequate for bending) ‘Thus, the section is adequate For deflection only Mathematics, Surveying, and 98 November 2002 ‘Transportation Engineering Seat Nas CIVIL ENGINEER Licensure Examination Saturday, November 17, 2001 (08:00 a.m. - 01:00 ps MATHEMATICS, SURVEYING, INSTRUCTION; Select the correct answer for each of the following questions, ‘Matk only one answer for each item by shading the box corresponding to the lettor ice on the answer sheet provided. STRICTLY NO ERASURES ALLOWED. Use pencil no. 2 only. ‘MULTIPLE CHOICE 1. Determine the central angle of a 350-m simple curve if the nearest distance from the curve to the point of intersection of the tangents is 18 m, ABP C36" Ba D. 56° 2. A reversed curve has the following properties: Degree Cer De Central angl Stato Determine the stationing of PRC (point of reversed basis A 67 + 505.23 ©. 67+ 467.23 B. 67+ 549.23 D. 67 + 689.23 3. Locate th ms : D. 2) 2 rent tata ie = ny ot sca tilt argh 0 enon ach eae led th walt hal Seaton cena 2 bois ante O68 bara Wits he present worth of a perpetuity of P1000 annually is money is WORN compounded monty? ABS? ssiform speed of #0 ppd. on the brake causing the car to. decelerate Unfors he distance from te ronlock io the pout where thescarsiopetis 12m. Find if the perception reaction time is 2 3m © tsaom B S543m D. 8765 m Civil Engineering 99 Reference Vol. 2 ‘8. Water flows into a tank having . Triangle ABC has side AB = 160 cm, BC= 190. the form of a frustum of a right circular cone, The tank is 4 m tall with upper radius of 15 m and the lowver radius of 1m. When the water in the tank is 1.2 m deep, the surface rises at the fate of 0.012 m/s. Calculate the discharge of water flowing into the takin m/s, A002 C. 008 B ons D. 012 Using a 25-m tape, a square lot was measured andl found to have an area of 1 hectare, Ifthe total error in area is 4.008 square meter short, what is the error in each tape length? ‘A. 0.005 m too short ©. 0008 m B. 0.008 m toa short D. 0,005 m. ng side CA. Determine the area of rangle ABC 1653 em Ct2dem 8 15i20m D,1739em A house and lot costing F2 million was bought at a downpayment of 50,000 and P1 mulion after one year. The remaining balance wil be past atthe end of te third year tke interest rate b 248, compounded seat annually, what is the required payment? ‘A. P?432.87800 © P1.27432400 “3 7215.00 low many combinations consisting of 4 digits can be made tsing the digits from to. hie Ais A210 c 120 B. 360 D. 420 Evaluate the integral of x cos (4x) dx with Tower limit of O and upper limit of a/4. As c 1/16 B. -i/a D. -i/16 |. An observer wishes to determine the height of the tower. He observed the top of the tower from A and got an angle of elevation of 3°. He then walked 25 m closer to paint B and observed the angle of elevation as 40°, Points A and B are at the same elevation, and on a direct line with the tower, How high is the tower? A. 21m ©. 45.27 m B. 5732m. D. 7632 m ‘The ground makes a uniform slope of 5% fom STA 12 + 180 to STA 12 + 240, “AUSTA'12 180, the center height ofthe roadway is Lif, At the ater station, the center height is 2mm ext ‘Hind the prod ef he find rod A 2 © 27% B isx Dae ‘The observed interior angles of «triangle and their corresperding number of observations sas oles ace Comer” Angle” No. ofobservation me 5 alae co c & Mathematics, Surveying, and 100 November 2002 ‘Transportation Engineering ost probable value of angle C a 48 56" vu. B ‘The cost of producing a commodity consist of PS5.00 per unit for labor and material cost and P25.00 per unit for other vaviable cost. The fixed cost per month amounts to P700,000. If the commodity is sold at P290,00 each, what is the break-even quantity? D. 3400 ertia of the area bounded by the curve x2 = dy, ‘with respect to the Y-axis, @ 524 DB. 25 ‘Acar travels on a banked circular curve. The front wheels of the car aze 1.5 curve is banked such that the two wheels are 15 em That should be the minimum radius of the curve so that sal pressure on the cat's wheels at a speed of 62 kph, ©. 3239 D, 2852: 20 em. Find the volume of the solid in ce lar to a given diameter Is an equilateral triangle Me ie58029 ©: 1547521 B. t94sna7 D. 1421398 Determine the area bounded by the curves x= I/y, 25 y= 0,4=.6,and the ‘A. 2138, hus house to his office. AL 21 km B18 kn Salve for the sum A,B, and C from the following equations: 2A’ -' 2B + 3C = 24 ‘Asolidstoo! ball of radius 3 cm is immersed in a cylindrical tank containing, ‘Water to a depth of 1D em. Ifthe water in the tank rises 2.25 em, what is the diameter of the tank? ‘A. Bem B dem ‘A circular platform of radius 2 m is revolved about its center at the rate of 8 ‘pm. What is the normal acceleration at the edge of the platform, in m/s"? A153 1408 Bm D, 116722 Civil Engineering Reference Vol. 2 101 26. The three sides of a triangle measure 36 cm, 18 em, and 24 cm, What is the 2 length of the modian drawn the longest side to opposite vertex 12398 em eC Mh en 811225 em D, 13.085 cm A cle of ais 9c is reumscribed about tangle whose area is 4823 square ci. If one side of the rangle measure 18 cin, determine length of the other side. 2 iy 2 ‘437m B 164m C 864m, D.17.09m The distances from the focus to the vertices of an ellipse are 4 and 6 units, Determine the second eccentricity of the ellipse. A. 0204 C0503, B. 0305, D. 002 ‘The sum of two numbers is S. What is the minimum sum oftheir cubes? A. 83/3 Cc $32 B. 5/4 D.S/5 Water flovrs atthe rate of 1o'm’/min in conical tank 12 m diameter on top and 24:m deep. When the wator in the tank is meters deep the surface rising a the rate of 0.566 m/min. Find the value of h 10m Cm B 14m D.8m “END ANSWERS: be 46 68 A 10 1D 2.0 we ZA TC ZA 1B RA RD 3D 8B 1B WA MD BA | ho 90 mC eB mA mB 5B WC BA wo mC Me es 102 November 2001 Solutions to November 2001 Examination ai ms ‘Mathematies, Surveying, and ‘Transportation Enginesting The nearest distance fom the curve to PIs the extemal distance, E ing of PRC = Stationing of PC. + Let 2+ 3642 # 3924 — 49(e2 + Be + 16) + 36) Gea, Wo oy @ 6y - 656-0 a= 09,227 B=36,b=6 ir= 2% _ 28" — 10.286 ree 1 = 60 oases 7 Gy + 9) = 656 + a9(16) + 36(8) = 1764 Reference Vol. 2 203 ell assume the shape ofa triangular prism, ime is V= Ay L, where A, is the base area with the shape of equilateral triangle of side 0.3m. (015)(0.15) sin 60°] x 6= 0.0584 me! ge mee ERtnmaty = ERvnty (peat = a+ 010/122 1;7= 010471 10,000 p= 20.000. «pos 0.10871, ee a7 From to B: (t= 3 see) a=-10 2,= 80 kph = 22.222 m/s eae gr Ye=BKch wnB0h vant 51= 6667 m From B to C Poneae O8= 2222%8+ 240) § Se= 24.69 m 5,4 5:4 12= 6667+ 2469+ 12 = 103.36 m as aus ane as 01258 Re1+s=140125h Th RE Rr v= Bowers v= Sa + 01259 + (+ 0.125H)0) + 1) ah v= Seca. F lo.se2sie + os7sie+ 34) 105 ‘Thus, the tape is 0.005 m too long, Ss Anes Adie *8¢100)(AF) 6% = 336(160)090) sip AE=1824 cm Balance = 2,000,000 - 500,000 Balance = P1,500,000.00 ne Nese cranes L | nee] ; 3 h ~ososas 7 Bt Pee smo Preheat’ ton x= 1500000 x= P1,387,215.00 Number of combinations = C(10,4)= fisesse Brom foas -so- fo umx; duedx; dom cosdrdr, 9=%sin4x ww Mathematics, Surveying, and ember ZOOL ‘Transportation Enginesring wats ox Zante. [sings a a Pp ake a eae [2S Desir] [iar bono eee ee a 6 = 180° - 40° = 140" (= 180° -30°- 140" o=10° In Triangle ABO: x 25 Bin 140° ~ inl x= 92512m A 2m 6 c In tiangle ACO: sin30"= 4 y= 92.542 sin 30" = 4627 m 015%) =am Sa. 12 +190 Sto. 12 +240 y73-28=02m day 14-02 , Grade of finished road = ~2—¥ 14" Grade of finished road =-2% Using the formula: Jons=c,] Gu" -002 = 26 Civil Engineering Reference Vol. 2 107 nn ie ‘The sum of interior angles of a triangle is 180°. ‘Sum of interior angles = 41° + 65° + 75° = 181° Error = 181° ~ 180" = 1° = 60° ‘The weight of error of each observation is K/N, where K can be any constant and N is the number of observation, The ideal value of K is the LCM of all the number of observations, Le, K = 30, ‘The error must be subtracted to the observed values since the sum of the observed values is more than 180°, Most probable value of angle C= 75° - 94.61" = 74° 25° 23.4" aw To break-even, Cost™ Revenue ftaN=pN "700,000 + (65 + 25)NV = 290N; N= 3500 mis 108 Noverniber 2001 _—_—"“tramspomtation engineering is oN on he whe tn = = ee v= 62kph = 17.222 m/s at eae 17.222? tans 73g = 22 981R 015m R= 30083 m v= 2po+aenam) +0 V=18,475.21 ce &y= 1.4142 , 1.4142) As=¥AO7071)(.4142) = 0.5 fe fee 70 Reference Vol. 2 109 am [ (Eos A= In (0.7071) = 2.138 an Tet Bf he bal ie sera in Se en sak He one Se et 2{l-8) 3/60) $= 2t-161/10 oa ‘Leaving at 830 and walking kp, he wil rive 6 minutes late, ss 5) + 6/60] . 0= Bt 126/5; t= 91 = 3(9:1) - 126/521 km O23 By Cramer‘ rule 2-2 a) 2 2] p=f1 3-2/1 3|-pe+12+13)-7-16-4 lp 4 ala. 4 D=37 2-2 3] 1 -a nan [-ts 3 -2[-15 3 =A! alee | Na= [216 8-180] ~ [18-192 90) =148 48 an Ma Mag 2% 3f2 2 Ne=|1 -15 -2] 1 —15 ja -2 3|a -2 Ny= [80 ~ 144 ~6]- [135 +8472] =-185, Np _ 185 aor. seen 110 November 2002 New en aa fone h 3 -15}1 3 js 4 -2)3 | Ne [12+ 90+ 96] - [216 - 120+ 4]= 74 Ne 74 eens G D 37 Sum=A+B+C=4-542=1 ig 100, Parad Imi 078 crim * “ree * BOsec o- © rad/s n= (4/15) (2) = 1.404 n/t Solving for @ by cosine law in triangle ABC: 242 = 361 + 18° 2(36)(18) cos 0 = 36.336° In triangle BCD: 22 = 188+ 182- 2(18)(18) cos 36.336" == 11.225 em ‘Mathematics, Surveying, and ‘Transportation Engineering Civil Engineeris Reference Vol. 2" 111 eter of the circle is 18 cm, hence one side of the triangle is the fof the circle and by principle tis tingle is a” RIGHT 2b= 96.46 20 etpae 27 324-B >Q) Squaring both sides of equation (1) b= 93045316 (24-19) B= so0asa16 1-324 2+ 9304.5316=0 suis (321° Re B=31.848; b= 5.613.0m a= 17092cm ‘Thus, the shortest side is 5.64 em Second eccentricity, & = 5 1 v= 1 mone a OTHER ELEMENTS OF ELLIPSE First Eooentscty, = © Eipseflaess j= £2 Angular eccentricity, a = £ Second flatness,7= “=P 112 November 2001 Ds Let.x and y be the numbers, then; aby Syy=S-x as+ya8+(5- 2 8-a)8=5/2andy=5/2 Sum = (5/2) (9/2)'= 9/4 16= re (0.566) hem Mathematics, Surveying, and ‘Transportation Engineering Reh eeateeiteie 113 Seat Now CIVIL ENGINEER Licensure Examination Saturday, November 17,2001 (02:00 p.m. - 06:00 p.m, SETA INSTRUCTION: Select the correct answer for each of the following questions. Mark only one ancwer for each item by shading the box corresponding to the letter of your choice on the answer sheet provided. STRICTLY NO ERASURES ALLOWED. Use pencil no. 2 only. MULTIPLE CHOICE ere Nearer iets Peak cialis fee caer ee eet eae eee ene cee an a sere Barletta eee wearers eee! ag ee ous, oes ; Bae ea ee eee en eeu tle ari eral eet te ral ter uari ome thabews oe eae roe 112,320 Ibs perfoot , 342/320 lbs per foot 4. A rectangular gravity dam 42 m wide and 2 m high has 20.4 m deep of ‘water at its upstroam side. What is the factor of safety against sliding. Assume 1. = 0.6 and use unit weight of concrete ~ 2510 kg,/n?. A 06 C08 Bo7 D.09 5. The two reservoirs shown in Figure 014 are connected with new 1200 m Jong of 900-mm pipe with roughness coefficient n = 0.012. After 10 years the Value of n= 0.024. What is the percent reduction of flow in the pipe. A. 50% ©. 75% B 25% DB, 200% 6. Determine the value of y in the manometer shown in Figure 015, ‘A. 0432 m, C0283 m Be 0.234 m D, 0324m 7. A 1.8m wide rectangular canal carries a flow of 2.4 m?/s, What is the citical depth? ‘A 0559m ©. 0.452m B 097m D.0378m 8 A480 ce soll sample taken from the se weighs 8505 grams. After oven drying, i weighed S944 grams. Ifthe spette grevig of solids s 272 dletertine the void rato ofthe sll 114 November 2001 Geotecunical Engineeeing 9. A 480 cc soil sample taken from the site weighs 8505 grams. After oven drying, it weighed 5944 grams. If the specific gravity of solids-is 272, determine the degree of saturation of the soil. A079 C0923, B 0979 D. 0.843, am is as shown in Figure 17. If the coefficient of permeability of the soil is k= 5 x 10° m/s, determine the seepage into the ditches per meter length of the cofferdam. ‘A. 0086 m/s © 0.104 m°/s B. 0072m/s 1D, 0.058 m/s 11. Water flows through a 25 cm diameter nozzle at the rate of 10.L/s. Calculate the dynamic force available at the nozzle tip. ‘A 13N. © 3a2N B. 276N D. 204N 12, Water flows through a horizontal nozzle located at the vertical side of a large tank under a constant head of 8 m. The nozzle is located 10 m above the ground. Determine the time for the water to reach the ground after leaving the nozzle. bl oreo Sad 2 ethane andrea eee Sea fii ate neva oat Uaeameecoes Mie min = . ed under a chamber ao ee aes fa enar ae inane ieee ee Bianca se caiman ae elie P3 cue. re cus = a 16, Civil Engineeris Reference Vol.2. 115 18. A semi-circular flume 1.2 m in diameter is used to carry a flow of 1.75 me/s when flowing full. If the roughness coefficient = 0012, what slope is require fd © ogo1a i Bons 5. boo 19, ‘The dichatge though » Cipolltti weir with eet length of & fect under a fend ef a5 Beis notes” a mechs © res B ibtes 52s |. Water flows through a horizontal Venturi meter whose inlet diameter is 31 manu throat dlametor is 19cm, The pressure atthe inl: s 755 kPa and a ‘A 07 is 550 kPa. Determine the discharge in m?/s. Neglect head lost C089 B 097 D. 062 ‘An undisturbed soil has moisture content of 54.2% and a Lquid limit of 56%. Determine the compression index of the scl ‘A. 0165, Come a B. 0231 D. 0567, 22, Soil mass consists of solid particles, water, and air. The -ombined volume of water and air is called voids and its ratio with the volume of solid particles is called the void ratio, ‘The ratio of the mass 0’ solid to the total Volume of soil is termed as C. moist unit weight B. dry density D. moist density 23, In wastevinter treatment the process of removing fine reidual suspended A. Aeration ©. Soda ash B. Filtration D. Zeolite Figure 015, Hydraulics and Civil Engineering 115 116 November 2002 Gcotecnnteal Engineering Reference Vol. 2 a 16. A semi-circular Name 12m in diametor i used to cany aflow oF 178 me/e wher flowing full “Ifthe roughness coeftcent n= 0.212, what alope is | required? | A. 0.0009 ©. 00012 t B. coos D. 00017 19. ‘The discharge through a Cipolletti weir with crest length of 8 feet under a \ head af 3.5 fect is nearest to essay ‘A. 176 els © 197 fs B. 154 eis D. 243 cfs 20. Water flaws through a horizontal Venturi meter whose inlet diameter is 31 ‘cm and throat diameter is 19 cm. The pressure at the inlet is 755 kPa and at the outlet is 550 KPa, Determine the discharge in m!/s, Neglect head lost. ‘A073 C089 B. 097 D. 062 | 21, An undisturbed soil has moisture content of 54.2% and a liquid limit of 56%. a Determine the compression index of the soil a aa A. 0165 C0414 Figure 013 B 0231 D. 0567 f ass consists of solid particles, water, and air. The combined volume ter and air is called voids and its ratio with the volume of solid ( Particles is called the void ratio. ‘The ratio of the mass of solid to the total Volume of sol is termet as: ' A. dry unit weight © moist unit weight } gly density B, most density 2 i 25. Im wastewater treatment, the process of removing fine residual suspen i solids known as r re er i ‘A. Aeration Soda ash B. Filtration D. Zeolite Figure 014 q Le Figure 015 Figure 017 116 November 2002 cotechant Easing RetecenteVol 1° aaa Solutions to November 2001 Examination Ax 6+ Y4O)(L428d)x2 A= 6+ 1A28e P=6+ 201.7434) Pe 6+ 3.8864 6+ 14284 6434860 ~ad pasin es P 5 (6d-+ 1.4284 6+ 3.4860 o013 ‘eas saned 643.4860 | a= or 189 2/3 } eam (6+ 1.4288) = 21.802 By trial and exror using the choices, d= 2m. Figure 014 [to ee ms ao [RRP END“ ‘Consider 1-foot length of dam: Ren Poyh A Rr = 62.4(60)(60 x 1) Re= 112,320 Ibs RaW Wie itd 1040 (yey = 24,601 118 November 2001 ms = 06(242,550) _ Fs, =07 208,080 HL~30m=hf y= 0270? Des Atyeard. 1029(0.012)4200)(00") | 9, a 09575 = 3.101 m/s Atyear 10: __ 10:29(0.024)7(1200)(2107)_ 5) _ ° esha. eet 80: Reduction of flow = 3.101 - 1.551 = 155 Percent Reduction = 155, 100% = 50% 310 Mydraulics and Geotechnical Engineering Reference Vol. 2 119 ae Sorming-up pressure head from Ato Bin meters of water: 5 Ps oe +408) +15-y03.6)= PE 98 oe ae v where po 0 y= 0324 m & = 24 ~ 11335 m/s per meter wi a= = 24 — 1.359 m/s per meter width (399)? an4 = 05659 m Sg 7 05659 Gscmec i esos wo WSS sage Wai epa ann Bi iy 05-504 nic = BOSS W, poco y= BREBMO «396 mn “te GxMC=Sxe G+GMC, ed ra BEBO) aa95 274.0) = 5.18) +0979 120 November 2001 io My@rantics and Geotechnical Engineering Ny 3am ws aut 7 Na H=38-18= 20m Ny= number of flow channels 3 9= Gx 10903 4= 0.083 m/s per meter Since there are two TImpeniows = 0.043 x 2 0,086 mi/s per meter P= Qy, g p= 2019810) 981 Af ta oO Sea a 20372 m/: © cies * (0025) (20972) = 203.72N From Yay tee 510=0- v9.81) @ (=1.428 see Q= Lass LHve | 24-1859 L 0.692 [=278m . hie * 6000) SHO | unsaem 2500= Civil Eny momrene vols 121 os Pro Yous haat 76 r= (20)(2) + 16(1) + 14(2) pr= 84 kPa For drained Condition, a3 = 90 - 40 3 50 kPa, Deviator stress, 61 -aa= 60 kPa 30 sind 5 o=22008° Q=Av=AL RA sia 175= $1.2) sur a 12/9)" 912; 5= 00017 = 8 feet (1m / 328 ff) = 2439 m H=35 / 328= 1.067 m (Q= 1.8591 #9 = 1,859(2.439)(.067)¥2 0-50mi/s Q=1763 ets iz AP. biases r= Bi Pe Oey 981 aT Q= 0.619 mys ‘Compression index, Ce For remolded soi C= Q007{LL - 10%) Forundisturbed soil C.= 0.009L1. - 10%) .= 0.009456 ~ 10) = 0.414 Civil Engineering 123 ‘Seat Nos CIVIL ENGINEER Licensure Examination Sunday, November 8, 2001 08900 am, -01:00 p.m. SETA correct answer for each of the following questions. ichitem by shading the box corresponding to the letter swer sheet provided, STRICTLY NO ERASURES ALLOWED. Use pencil no. 2 only ‘MULTIPLE CHOICE Situation 1 =A cylindrical drum 2 m in fads is held by a rigid bar AB hinged ea and a Aexbe wire cable BC ay shown ia Figue WES. ‘The Caan ‘weighs 1500 N- Neglert fcon in surfaces 1. Wiehe lowing eves them 3 3500 2. Which of the following gives the vertical reaction at A in Newtons, A 1468) C159 B 1853, D, 564.2 3. Which of the following gives the tension in the cable BCin Newtons A. 27683, C2439 B. 22554 D. 2854.1 Situation 2~ A reinforced concrete rectangular beam has a width of 300 mm and fective depth to bottom bars of 450 mm. The beam is reinforced with 2mm bottom bas and two 28-mm top bars located 65mm from the fp of the beam. Concrete strength f;= 345 MPa and ste! strength fy = A Wh Ha flings in Fete rca cas of ts Senn section in percent. au 5 72% 6. Which of the following gives the nominal moment capacity ofthe beam in KiloNewton-Meter. ae ‘a. 5708 42 5 7436 D. 4238 Situation ~The inplementing rules and regulations of PD, 584 states that for ity overs or undermine 3f nok more than seructural Engineering 124 November 2001 and Construction Tot be covered by a Change Order provided that the same is authorized by the “Approving Authority for the contract. ‘The quantities and contract prices fora project are as follows: Quantity" UnitPrice (pesos) Remt = 300 70 Tem 2 0 500 Kem 100 0 em 150 00 Tem 200 0 em 20 500 1 owing gives the total contac price ofthe project: 2 Peabo00 i, patnjoon s the following gives the minimum price af a item in onder eniajorpayion 2, i C. P185,000 Piss) D, Piat.000 9. Which ofthe folowing items is considered as a major pay item ‘5 hems © tem? B tent Do tem 4 | Situation 4 - Section 37 of NSCP tates the following for simple timber sli tolonn desig Allowable wii stress in N per square mm of crose-sectional area of square os rciangular simple slid cofum shall be determined by the following, fens but such ut stress shal not exceed values for compression, parle to grain in Table 3. Short columns (le/d of 1 or E= modulus of elasticity, MPa b= effectivespan length = Ke, Civil Engineering Reference Vol. 2 125 ee ‘AT10 mam > 140 mam Apitong 3 m longs used asa column. The column js ngs at oh ens with Kw. The properties of some Plippne woods at 00% strew grade areas fellows: Table 31 ‘B08 Stes Grade Modules) [Comper | Caprese] — Shar isecityin | paielto | perpencicdar| pall © beng | gan Baan | gain 10. Which ofthe following givés the classification of the column. A. Teng column 1. Ww si compression parallel fo grain adjusted for L/d ratio, in MegePascals, A 56t aa . D478 12, Which of the following most nearly gives the axial load capacity of the column in KiloNewtons, 13. Which of the following gives the acceleration of the truck in z/=2 ALLS © 197 B. 185 D. 215 ving gives the minimum cooffcie friction between the ures and the will prevent the truck from sliding while itis at rest ‘A. 005 ©. 008 B. 013 D, 028 15, Which of the following gives the normal reaction at the front wheels of the truck in Newtons, ‘A. 9432 c. s220 B 10348 Deo Situation 6 - Section 595 of NSCP states the following about control of deflection. 595.1 Reinforced concrete members subjected to flaxure shall be designed to have adequate stiffness to limit deflections or ary deformations 126 November 2001 Structural Engineering. ‘and Constrvcsion that affect strength or serviceability ofa structure adversely. 5952 One-way Construction (Nonprestressed) 59521 Minimum thickness stipulated in Table 393 (shal apply fc one-way construction not supporting or attached to partons or other yt be damaged by large deflechons, unless computation a leat hicks can be used without adverse effets, ~ MINIMUM THICKNESS OF NON-PRESTRESSED BEAMS ‘OR ONE-WAY SLABS UNLESS DEFLECTIONS ARE COMPUTED * “Minsnwan Disks Simply | Oneend | Bothends Cantilever” Pret supported | continuous | continuous ‘Members not orattached to partitions or other ‘natracton Hkely to be damaged by lage deflections Solidonewayslabe | L/20 | L/a | L/2s £/40 ‘Beams or bbe eee bye | ones | Lyn Ls 16. v length Lis in millimsters ‘hall be used diectly for members with normal density roncret ‘and grade 415 reinforcement. For other conditons, the values shal lowe ighnwoight concrete having unit weights in he range 150-2000 g/t the values dl be map by €£65- 008 i) butts than 3.09 ® (ost g / 700) ‘The section of sold one-way slab s showin in Figure CO.97. The concrete is a normal density concrete with @ unit weight of 2350 kg/m’. The reinforcing steel used has a strength of fy 276 MPa, ‘Which of the following gives the minimum thickness of slab By in suillimeters, © 318 D, 329 Which of the following gives the minimum thickness of slab B: in sullimeters. Reference Vol. 2 127 ‘haft 60 mum in diameter and 65 m long is subject to sre ME-29, The shaft is attached to a rigid support. ‘Modulus of rigidity of steel is G= 88 GPa. 19. Which of the following gives the reaction at the rigid support in N-m, ‘A: 00 C1200 B. 200 D. 1000 20. Which of the following gives the maximum shearing eress in the shaft in MegaPascals 2829 2036 B. 2543 D. 2358 21. Which of the following gives the total angle of tvil at thé free end in degrees ier ‘Situation 8 - The labor anid material cast for a riprapping werk is as follows: Labor: 11 Foreman @ P50.00 per hour 1 Skilled worker © F'40.00 per hour 5 Unshilled workers @ P39.00 per hour ‘Materials (For every 1 cu. m. volume): Portland cement = 3 bags @ P150.00 per bag Sand = 0.25 cu. ma @ 7300.00 per cu. m. Boulder ~ 1 cu. m_@ P250,00 per cu. m, ‘The accomplishment of the above laborers is 0.45 cubic meter per hour, Profit, overhead, and miscellaneous expenses is 30% of the labor and material cost 22. Which of the following gives the labor cost per cubic mer, ‘A. P265.00 ©. 678.00 B. P589.00 D. P521.00 23. Which of the following gives the cost of materials per cubic meter. ‘A. P565.00) *C. P7500 B. 865.00 D, P655.00 24, Which of the following gives the total cost of riprapping per cubic meter, A. PI,83200 C. P1,982.00 B. PL 75400 D. P1,635.00 Situation 9 - A propped beam is as shown in Figure AN‘, The moment ‘applied at the simp supported end causes aunt rotaien at that end 25. Which ofthe following gives the value ofthe moment Min KN-m. ©. 600 A 60. B. 550 D, 500 26. Which of the following gives the reaction at the simple support in KiloNewton. A875 ©. 10455 B. 7585 D. 9375 27. Which of the following gives the moment at the fixed end in KN-m, A. -250 © 150 B. -450 Di. -550 Structural Engineering 128 November 200i ‘and Construction Situation 10 - Section 451.4 of NSCP stater the Following for bending oF Taterally unsupported beam 1. Tension: Fy~ 0.60 F, 2 Compression: ‘a, For members meeting the sments of Sect. 4.91.2, havi the plane of their web, rs of channels bent about their ser value computed by Form (452), a applicable (unless a higher be justified on, the basis of a mare precise anal ‘more than 0.60 Fy * (45~6a) Gre es Ee (45 - 6b) Or, when the compression flange is solid and approximately Fectangular in cross section and it area isnot les than that of the tension flange: yn 83.0006, ad/4;) (5-7) Sn the foregin 1 istanee been cross section braced against twist or later of the compression flange, mun. For canlevers braced aginst the support may conservatively be taken as Ue actual length rr radius of gyration ofa section comprising the compression ange plu’ 1/3 Of the compression web area, taken about an ans in the plane of the We, ‘Ay ates ofthe compression ange, mm? fe than 23, where Mr is the ire bending). When the bending moment igth is larger than that at both ends of be taken as unity fir frames y conservatively be taken as unity for W21 x 62 steel is used as a beam simply supported over a span of 8 m. The beam is laterally unsupported over Radius of gyration, 77 = 53.34 mm Section modulus, S,= 2,077 x 108 mm’ 28, Which of the following gives the value of the ratio Lr A130 C170 150 D, 160 of the following gives the allowable banding stress in MegaPascale in ‘© with the provisions of the National Structural Code of the © 201 30. folk eet distribut 20. W lowing given the safe efor datibut Toad thatthe beam can cry in cron pr meee € 1656 Bisa Dise 7200 tern 1000 Figure ME-29 Lean E= 200000 Ne 12> 10° mm Figure AN-16 130 November 2001 a eariateesen Civil Engineering a Figure co-37 ee 7464 m/f 19 irom the force polygon in Figure (2) END“ sealer erase OF sina = 2S; Ry=3000N Rp Foe () i, = 36.87" na 6 = 0-30°=6.87 [EMa=0] (Tacos a)(10) = Ro(7.464) s AN EARERTSE oe la uc cos a)(10) = Ro 4C @¢ 10 OC MA HO (Tac cos 6.87°)(10) = 3000(7 464) . ZA TA BC A 2B WA Tac = 2255.39N 2b 85 Ge we AC me id Be uc Wo MA wd [EFv=0) Ray + Tyesin@= Rosin 30° SA WA HO mo wc mC = Ray = 3000 sin 30° ~ 2255.39 sin 36,87° Ray= 146.76 = 450mm 0.008 (200800 I yiede f.1200,000 _ 108 -=374.6MPa >hO ‘Thus, the depth of compressi Nominal moment cacy Civil Enginees Reference vaiae 133 ‘Situation 5 (709) v6] 250 0 152% Toul ‘Total contract price = P790,000 ‘Minimum price to qualify as @ major pay item = 20%(790/000) = P158,000 Among the items given Item 2is the major pay item ng si y {situation (100-12) F.= 9.56 MPa (from Table 3.1) E=731(1000) = 7310 MPa (From Tabl = F3i(1000) xeon [E =os7 =1855 . z 956 Since L/ a> K, the column is a Long Column. = F-A=4.775(140 x 140) ~ 99,5951N = 93.595 kN © Situation 5 13 to 15) Structural Engineering ‘and Construction 134 November 2002, ‘Receleration: vane + 2aS = 60 kph= 16.67 m/s (16.67)? = 08 + 24(75}ra = 1.852 myst tan 0 = 8% ~ 0.08; 0~ 4574" ‘Minimum coeffi ‘when the truck is at b= tan = 0.08 friction J-0 FOS) Note: REF=M xa 4.574") ~ 1800(1.852)(05) 1 Situation 6 (16 to 18) ‘The concrete is a normal weight conerete, Since the steel grade used is not grace 415 MPa, the value shall be multiplied by (044+ fy / 700). © Forslab B: (Cantilever) L — £000 gy (as f / 700) = SO? (a4 + 276/700) 318. mm For slab Bx: (Both end continuous) L 8000 Tinin= (04+ f, (700) = "2 (0.4 + yp OM / 700) = = (04 + 276/700) Thain = 227 man For slab Bs: (One end continuous) eee ods, = 2200 (0.44 276/71 Tia = 3, (A+ fy (700) = “= (04 + 276/700) Pious 199 mn Civil Engineering 135 | Reference Vol. 2 ‘Situation 7 (19 to 21) Reaction af the fixed ond: {ET=0] T+ 1000= 1200+ 800; T= 1000 N-m Resultant torque in each segment: Typ=-T=-1000 N-m ‘Toc = 1000 - 800 = +200 'N-m Ten = -800N-m Maximum shearing stress: = 1000 N-m Angle of twist at the free end: . te 5- je entt 5 Di = (60)! = 405,000 mam? ‘ ° [-1000«10%(2000) + 200.109(1200) * 05 000%(85 000) © @ = -0,03882 radians x * 136 November 2001 DSituation 6 (22 to 24) P265.00/ hour (OA5m§ /hour Labor cost per cubic meter = Structural Engineering ‘and Construction ‘hour = 50(1) + 40(1) + 35(6) = P265.00/howr = P588.9/eu.m, Material cost per cubie meter = 3(150) + 0.25(300) + 2(250) Material ‘cubic meter = P775.00/eu. m. vat 388.9) = P5889 é bic meter = P1,831.96/eu. m. AA Situation 9 (25 t0 27) ‘M= moment in kN-m R= reaction in KN = 1 radian ‘and miscellaneous = 30%(588.9 +775) = P400.17 bic meter = P588.9 + P775 + P409.17 + P58.89 16R/3~ AR = 125; R = 93.75 kN M=16(93.75)/3 ‘M= 500 kN-m Ma=M-8R (Mg =-250 kN-m_ (16/3) = O:M=I6R/3 > 2) Civil Engineering Reference Vol. 2 137 TD Situation 10 28 to 30) L=8m = 8000m L pee = [22000 - 1187 = 20 ut not more than 0.6 Fy. 217010", _ 117010") ny ~~ (9987 _ 83,000¢, (a7 Ay)” #90000) Fa000(s33)/3276) 27 also 06 F, = 0.6(250) = 150 MPa = 5201 MPa and Fy Ag= by y= 210(15.6) = 3276 mm: 7 ‘MPa Thus, Fs» 63.77 MPa Uniform Toad: ‘Moment capacity: M 5. M= Fu 8. = 637702077 x10) = 152,450,200 Nm M= 124510 pe 22) apes = 28) 3 5 w= 16.56 kN/m_ Beat Nox CIVIL ENGINEER Licensure Examination Wednesday, May 1, 2002 £600 am.- 01:00 pm MATHEMATICS, SURVEYING, AND TRANSPORTATION ENG'G. SEA ct the correct answer for each of the following questions, Mark only Dy shading the box corresponding tothe lelier of your chose oe pencil no, 2 only MULTIPLE cHoIcE 1. Bvatute tn [E228] si D, infinity Which of the following has the greatest effective tate? A. 1231% compounded quarterly. B. 12.20% compounded monthly CC. 1285% compounded annually D. 12.32% compounded semi-annually = 6nPs, what isthe value of n? G1 _D.6 ‘A carcle of radius 8 cm is inscribed in'a sector having a central angle of 80°. What is the area of the sector? © s2a7em? D. 475.42cme Given a triangle ABC with sides AB = 30 em, BC= 36 cm, and AC = 48 em, Find the distance of the point of intersection of perpendicular bisectors 9 side BC, ‘A. 1592cm B. 18.21 cm D. 10.47 an ‘A baseline measures 25 km at elevation 520 m._ If the average radius of ‘curvature is 6400 km, compute the sea-level distance, A. 24,996.24 m_ © 24,997.97 m. 5B, 24,998.65 m D. 24,995.24 m Determine the distance from the curve x+y! 1éy + 16r + 64 = Dto point 4) me iverted and the other is upright of 12m and height of 48 m. Each cone contains eal sof ol ung pectic evi of 8 UF the dep of oi ihe Inverted cone is 24 me whats fin the uprigt cone? dese en ane 5 osm D.ozim 140 May 2002 % 10, ‘Mathematics, Surveying, and "Fransportation Engineering TA piece of wire having a total length of 72 em was cut into two unequal segments and bent to from two unequal squares. If the total area of the squares is 160 sq. cm, what is the difference in the lengths of the two segments? ‘A 24cm © 32em B. 16cm D. 28cm “The impact factor of a road with rade 500m i 015. What maximum velocity can the car travel round the curve. Neglect friction between the Toad and the wheel 4. 7.65 \ph © @251gh 8 12478 ph D, SLidiph particle moves asconding to the parametric exits yene fee where. x and y are displacements (in meters) in x andy dlretidn, epee and 1's ume In seconds Determine the aceeation of the jrafler f= 3 seconds © ma7mye D. 5.21 m/s? Determine the area bounded by the curves = dx and y= 8(¢-1), “A. 5dhsg units Arr aq. B. 23 oq units D. 474g, units From Station 12 + 180 with center height of 1.4 m in fill, the ground makes a uniform slope of 5% to Station 12 + 240 whose center Reight is 28 m in cut. Bit ‘point of intersection of the ground and the road from Station 12+ 180? A Bm 15m B 20m D.30m Bese erase eee ear Bn gon ast aa rc oes aes c rasan ene peace ofA square meter Ifthe height of the tak 15 hess bose Sear ieee B25m D.a2m ‘The volume of a regular tetrahedron is 3.182 cubic meter. Find its total surface area ‘A. 1247 99. m. © 16.8759.m. B, 1559q, m. D. 10.42sq. m. a 141 fehigh and 10 ft across the top is flee with a liquid that ‘weighs 624 pef. How much work is done in pumping all the liquid at the top of the tank’ ‘A. 8 the interes rate is 10% anally compounded quart, what hanp sum value can he expect after 20 year? 00000 30000 te area consist of a rectangle with ve ‘nd (3) and a trnngle with vertices at coordinales iow fat from the Y-axis isthe centroid ofthe compe 62st D. sar ing through (6, 3) has a slope of 3/2 Find the distance, slong he in, om the pot, ) 10 the an 5 542 D, 901 ‘What is the distance between the vertices of the following ellipse? Gt Boy Tox Tey GHB = 0. A 63: ©. 1021 B. 1254 D, 5.105 ‘A triangular piece of land has one side measuring 2 km ‘The land is.to be divitled into two equal areas by a dividing line parallel to the given side, ‘What is the length of the dividing line? A6 Peete striae reac eae, econd:-quurter point ofthe spiral, 20am - B. 677m © 199m, D, 358m How many diagonals have an undecagon? A 44 C. B. St D. 35 Determine the shortest distance from point (4,2) to the perabola y2= Bx. A283 C. 241 B 350 D, 632 Mathematics, Surveying, and Civil Engineering 142 May 2002 ‘Tramsporcstion Enginecring Reference Vol.2 143 29. ar ‘of the numerical eoeficients in the expansion of (2 + b)8. =+ Solutions to May 2002 Examination Bed i. oa 7 a es le numero and domino pproahe, we cen ry an Arr nes x 3a” a fe ale ld etcitnter nsec pes hn 2 2-208 Soc Z a 22 25% tim 59° a cs iene x the numerator and denominator approach zero, Sfeec* 2x(2sec? 22)-+ tan 2x(4see2xsec 2x tan?x)}+2c0sx = 0,001 radian (0:18/ degree) oa oe tan)2(0:18 / x)]~2sin(0.18 /x) =2.9999 ERS (1+ 4/myr-1 ~1~ 01289 = 12.89% = 1201291 = 12.91% n-B)=(n-3)(a~ A)in= 5). BREE Mathematics, Surveying, and ‘Transportation Enginecring. 144 May 2002 eo 7n+a=6 n2-7n+6=0; (n-1Y(n-6)=0 n=1 (absurd); 1=6 -x(20.4458)? (80°) 340" Arr = 291.84 em? Arter = ms ‘Note: The point of intersection of perpendicular bisectors of any pe See. SO) «24.03 em From the right triangle shown (shaded triangle) 32> RO~ 182 = 24.032 - 182 x=1892em By principle: j= horizontal distance atan altitude af (above soa level), m Civil Engineering Reference Vol. 2 145 D= Actual or Corrected distance at Ee 24.9797 ken = 24,997.97 m_ Using another formula (approximate) r= 1-H/R=1~052/ 6400 tor = 0.99991875 ce = 25,000 x 0.99901875 = 24,997.97 m ‘Mathematics, Surveying, and ‘Transportation Engineering Ay + Az 180 eGn) 7 2 art) T-x= 48 & 4 Difference in longth = 48 ~ 24 = 24.em Impect factor (centrifugal ratio) = tan = aR o15= 9.81(500) v=Z1125 m/s 0= 97.65 kph o- (ar = fae When #3 se: a= (36@)F +16 = 18.44 myst A=4 GV8)Q- ¥ 2VB)Q) A=377 99, units cal a Ne ave Mathematics, Surveying, and ‘Transportation Enginecring 148 May 2002 an T=321-T T.- 172.07 m Ta= Re tan (1/2) 17207 = Ratan (62°/2) R= 28637 m gg = 228637), Dee avecte(i-2] 250 ayeermmce(s-2) «ose Formula: Asie = @V3- Work= force x distance WWedPxd aF=y dV = 624 nx? ah d=12-h In the figure shown: x= 5h/12 dE = 624n(5h/12) dh P= 34034 hah 0 ~ 34.034 12 dex (12-1) aV7= 34.034 (1282 - JP) wre skoos fame! W= 5881075 1b 150 May 2002 ‘Transportation magineeing Tansther Salton me Work =Forer» Distance lox ooo = Weight ee oe Fore =7 V=624x $x6)(12) ] Force = 196034 hs Distance = distance from centroid 12 of volume to point of exit Distance = (12) = 3m Work = 19,608.54 (3) =58,810.6 fb problem with A = P6,000.00 every three months er i= 0310/4 = 0.025 6,000{(1 + 0.025)" -1] 01025 F=P1,490,296.28 FS 29-90 ale 3) Pee ARS Ns 7 Go) 6.0) 2A = 18(6) + 6(19/3) e F = 6083 : 151 Slope ~ tan 0=3/2 @=5631° From the shaded triangle shown: cos 0= 5/d d=5/cos8 et 5 aaer a-9018 Gry seiecetaeies 4 fs Sg @=9o18 Distance between the vertices = 2 dx? + 25y2-4 Tox - 16y - 648 =0 6x2 + 16¢ + 25y2 - Loy = 648 64(x2 + =/4-+ 1/64) + 25(y* ~ 16y/25 + 64/625) = 648 + 64(1/58) 64(x + 1/8) + 25(y -8/25)°= 651.56 41/9, y-8/27 01806” 35.0624 a= 26.0824 2=5.105 Distance between the vertices = 2a = 10.21 + 25164/625) divide by 65156 YN ‘Small triangle waN Big eengle 152 May 2002 m2 Mathematics, Surveying, and "Transportation Engineering ‘Since the small and big triangles are similar Arps Av aed = 8.485 ken Flow, = density x space mean speed Density = 14 vehicles / km 1ce mean speed = 30 miles/hr x 1.60975 km/ mile Sface mean seed = 48293 Kn be Flow,q= (A-vehicles/km)(48.293 kan/hr) Flow, q= 193.17 vehiclesyhour H=Dcosd p-Lsegeg De 1008) +03 D=1803m H=1803 cos 4°30" H=17974m 2B Offset distance to any point L from PC, x= Ly= length of spiral = 100 m RL, ‘An UNDECAGON is a polygon wit Number of agonal, D= 0 ‘Number of diagonals, D = 4 (11-3) =44 1 sides (n= 11) Reference Vol. 2 153 2s Ba (54+ (y-2F r= 4/8 dam (ye/8~ 4p + (y-27 eae a= Eps 164 yay aq Vt es aye =F aysn a” Differentiate é set d/dy to zero: ad 2 a ay pro yo4ands=2 Yano ro the variables. a ‘Sum of numerical coefficients = (1 + 1)°= 256 154 May 2002, een scmeeence vununeaiains 155 SeatNos 7. Which ofthe folowing is closest to the value ofthe factor of safely against CIVIL ENGINEER Licensure Examination rumeer Wednesday, May 1, 2002 62:00 pan. -06:00 p.m. B 154 8 Which of the following is closest to the value of the facior of safety against ‘ overturning HYDRAULICS AND GEOTECHNICAL ENGINEERING SETA ad S ¥ 33 INSTRUCTION: Select the correct answer for each of the following questions. 9. Which of the following is closest to the overturning moment acting against Mark er foreach item by shading the box corresponding to the Letter the dam in KN-m, of your choice on the answer sheet provided ‘A. 358.16 KN-m 285.63 kN-n STRICTLY NO ERASURES ALLOWED. Use pencil no, 2 only. B32412kN-m D. 39875 kNom MULTIPLE CHOICE S Situation 2 ~ Water flows froma tank through 160 feet of 4 inches @lameter pi ‘and then discharges into air as shown in Figure 20. The flow of water in 1. Calelate the magnitude ofthe tal hydrostatic fre ating on one side of wo. Ree 2g Auman “O03 and niet minor ne: smu square plate shown in Figure ichol wing gives the velocity of flow in the pipe in fps. Sa N ©. san4 KN Aes Csi fe B. 3652 KN D. 41.62kN B. 2 D. 1587 #/s 2. Two plezometers are installed on a tank as shown in Figure 17. Determine TL Which ofthe following gives the total head los in the pipe in feet. the value of i AL 7350: © 610 fect ‘6752 ©. 415m B. 5890. D. 6370 foot 5, 425m D. 435M 12. Which of the following gives the prssure atthe top ofthe tank in psi 3. Water flows atthe rate of 350 cfs ina trapezoidal canal having base width of AL 2827 psi © 8785 pa 12 fect and side slope of 2H to IV. Determine the critical depth, Assure n B. 2458 poi D. 1.745 psi =00015and $= 0.002 ‘A 2564 feet ©. 2312 feet Situation 3 - For the pipe system shown in Figure 17, n = 0015 forall pipes and B, 2968 feet sh mooie STH ri She fow inpipedis i ey eae 4. A tiangular channel with most efficient proportion discharges water at the hi following is closest to the head lost in pipe “in feet. fate of m/s. Assuming n = 018 antl $= O02] calculate the normal Caaayy ta ‘opt of flow in meters D, 339 0763 ©. 0632m lowing is closest to the total head lost in terms of the total 8. 0913m D.Li2dm isin cf 5, An open cylindrical steel tank 24 m in diameter and 3 m high contains 2.1 C. 0373292 ‘im deep of water, Determine the stress in the walls of the tank near the D, 0.4752 02 bottom if the wall thickness ie 8 mm. 15. Which ofthe following is closest to the total head lost in fet ‘A. 309 MPa ©. 357MPa A5374 C 69.63 Bl 247 Mra D, 274 MPa Bo 3541 D. 4724 6. A.rectangular suppressed weir of length 1 m is constructed or cut atthe top ‘ of a tall rectangular nk having a horizontal section 20 m by 201. Ifthe Situation 4 The soil shown in Figure 25 has avoid ratio of 050 and G=2.70. initial head over the weir ss 1 m, compute the time required to discharge 72 “15 mvin=3m. um, of water. Use Francis formula 16. Which of the following is clasest to the effective unit weight of sand in “a” 3652 seconds © 58,74 seconds KN/am B 4835 seconds D. 24:12 seconds Av1528 © 425 D, 112 SITUATIONAL 17, Which the flowing i closet to thefts stresa pont Anka 7841 Situation 1 - The section of a concrete gravity dam shown in Figure 37. The B 1864 Dar depth of water at the upstream side is 6 m. Neglect hydrostatic uplift and 18. Which of the following is closest to critical hydraulic gradient of sand (for ‘use unit weight of concrete = 23.5 KN/m?, Coefficient of friction between quick condition), ‘the base of the dam and the foundation is 0.6. 156 May 2002 Geotechnical Engineering rs ce sei en cnt eo Ti and greene ae ee ‘The ground water table is 10 feet below the ground surface. The angle of aera a 19. Which of following eer Ts < nae Fe arn EE te ay feet a pn from the ground ‘in foot. ee 21, Which of the following is closest to the overturning; moment caused by the active pressure acting on the wall per foot width, ‘A. 120,215 feb C, 150,687 fab B. 110.452 feb 1B. 98,458 fel Situation 6 ~ A solid concrete retaining wall is shown im Figure 67. The fill ‘behind the wall has a unit weight of 110 pef whose active soil pressure may be assumed equivalent to a fiuid pressure of 30 psf per foot. ‘The passive Beste may be assumed aquivalent to a fu pressure of 300 pf pet fot, live load surcharge behind the wall is equivalent to an additional of 2 feet of fil Assume unit weight of concrete = 150 pct, Consider 1 A length 22 Whe ofthe following ves the total active presse acing on the waln ‘A. 4500 ©. 720 8300 DB. 6000 2B. Which ofthe following gives the overtuming moment about the oe in fb. ‘A. 52000 ss C4000 B. 68000 D. 36000 24. Which of the following gives the factor of safety against overturning, AAD C241 ee ae J Tepe oie es san Og Faekcra eel a i 2 aE aac eee ee wr C4350 cree wg es pc A ce cr ee : i a Civil Engineering Reference Vol. 2 157 27. Which of the following gives the vertical effective preseure when there is ho ‘water in the sand layer, in pf ‘A 6710 c 530 8. 7480 D. disso Situation 8 - Inthe syringe of Figure 912 the drug had p = 900 kg/m? end y= Ga Pas The Aow tough the needle is Of t/a, Neglee head los in the larger cylinder 28, Which ot the following gives the velocity at point Bin n/s 7505 sia 5. 6.9 D. 6823 29, Which OF the following give the Reynolds number fr the flow inthe A917 . 1025 B 825 D. 167 20. Which ofthe flowing gives the stad force F repre to produce the a 155N © 243N 8 107N Basan Witz abe Ground sutace 2 135 nf fed en sot 158 May 2002 Sucharae Figure 17 Figure 37 Civil Engineering Reference Vol. 2 159 4000 18in. 1500 8-28, 20008 2¢in x ry sano 12in Figure 17 eee pesnea seexek Becca 160 May 2002 Solutions to May 2002 Examination 4 1sin 459(2 « 2) tes BE s075 +225- ham HO i f= 4.125 ma gv Atrial stage, 2° = (#28) x 2= 124+ 240 322 a+ad By trial and error using the choices, d = 2564 feet = 380.348) Hydraulics and Geotechnical Engineering Civil Engineering nowrenes Volt 161 aa Note: The proportion for most efficient triangular sectien is that of a 90- degree triangular V-notch. 162 May 2002 D Situation 1 (7109) Fe yah A=9316)6%1) F=176584N y= 0/3)6)~2m_ OM=Fxy= 176582) OM = 358.16 KN- w= 4-492) =3 m= (2/: = 1333 m Ra P= 17658 kN Rea Wi + We= 376 + 18 = 56LKN “BR | 06(568) _ _ 55,2 Ae » S908) -ane RM = x, + Wi 22 = 37603) + 188(1388) =1978.604 KN RM _ 1378.604 I oul: meg 7 2 Situation 2 (10 to 12) (Q=128/s=03401 m/s D=4" =0533 f= 101.6 mm 2 $(0.333)" = 137.785 ffs 10298 Lg? _ Bur 5 HIL= 194239 m = 6370.86 feet HL= civil Engineering Reference Vol. 2 163 ‘Energy equation between A and C: Ey-HL= Es Py Ph tap pa Bg Paz BEA Bay 0+ 2 +10-carae = 13725) 7 21822} +0400 2 = 6755.65 feet of water Y p=421,5528 psf = 2,997.45 psi Using the English units for Manning's Formula os 2 Raa sy v= 137,785 ft/s R=D/4=0333/4 = 0.0833 ft S=HL/L=HL/160 1.49 jeer 4 a 1B7785 = Ses (0.0833) (FIL / 160) HL = 6356 feet ©D Situation 3 (13 to 15) Q= 01-04-12 f8/s Mydraulics ond Geotechnical Engineering 164 Way 2002 ‘Total Head Lost, HL = Hla + Hla + Hy 14.637 (0.015)? (4000) 0.76720)" ‘Hg SSBB) NO) O.ZO7 EIS e717 HL 010888 Q? + 0.2826 e+ 00518 Q? HL=0,3732. 02 ‘Total head lost = 0.3732(12)? = 53.74 feet = 0.2825 @ @D Situation 4 (16 to 18) Effective unit weight = Submerged unit weight, 7» GAL, = 27-1 9g) 11.118 kN/ Tee” 1505 we Critical hydraulic gradient: cn ieaared Tse 1405 in =1133 Civil Engineering maces aust ig! 2 Eh AOS Ostia ; rae a reer tact “cra aati Considering 1-oot length of wall: Fressure =110(0.271)(10) = 298.1 pot = 57.5(0.271)(16) = 249.75 pst = 62.(16) = 998.4 pst otal pressure: 10) = 14905 Ibs F= 1490.5 + 4769.6 + 1998 + 7987.2 F~ 16245. Ibs Location of F: ‘va= 2/3)(00) = 20/3 + 1996(20.667) “+ 7987-2(20.667) Y=18.6ft 2=26-186=7.4it ‘Overturming moment= Fz = 16,245.3 (74) ‘Overtumning moment = 120,215.22 f-1b Hydraulics and Geotechnical Engineering 166 May 2002 D Situation 6 (2210 24) 2 ef ‘tf An f M ve ane reed] » PB cel! [aR tan or ‘cvs presse BR cope p= 300) = 60 pot P= 30020) ~ 600 pat ‘Active pressure: F,= x20) = (60(20)= 1200 bs By =¥ep, (20) = 32(600)0) = 6000 bs Total active pressure, F= F,+ = 7200 Ibs Overturning moment: i= 20/2=10ft v= 20/3 ft OM=Fis+ Fy ‘OM = 1200(10) + 6000(20/3) = 52,000 t-1b Righting moment moment: = 180(18 x 2 x1) = 5400 Ibs ‘= 150(12 « 2 1) = 3600 Ibs 110(20 x 6 1) = 13200 Ibs Ima Vx + Way» 54006) + 36006) + 13200) RM= er oo fe Reference Vol. 2 167 ‘RM _ 167,000 Factor of safety against overturning = Y= See ‘om ~ 52,000 Factor of safety against overturning = 3.22 © Situation 7 25 to 27) Vertical effective pressure at A before lowering of the water table: pa=Dyph= (120-~62.4)(12.5) + (135 - 62.4)(50) Pa= 4350 pst After lowering of the water table: Yu 116 + 0.2(135 - 116) = 119.8 pot a= (120 - 62.4)(125) + (135 - 62.4)(25) + 119.85) a= 5530 psf Vertical effective pressure at A when there is no water in the sand layer: pa= (120 ~ 624)(125) +119.8(50) Pa=6,710 psf Hydraulics and Geotechnical Engineering 168 May 2002 ‘DSituation § 25 t0.30) 2 _ 04x10 a= 8.1897 m/s - 8 = '8.1487(0.00025)(900) aura Re= 91673 (laminas flow) her Egutn betveen on -hi= DOA Par PEE Ey 2g Y 23 z 24=0 (negligible) Hb Wyinthe needle “The head Jost in the syringe is negligible Since the flow is laminae = 6i/e~ 64791673 =0.0698 x s826(0.0658(0.02)(0.410-2 (0.00025) HL=1889 m 7 _ 6as87) + FA 40 1889= Or 20981) pa= 196681 Pa +040 ‘Force, F= pax Area of piston Force, P= 196681 x £ (0.012=15.45N Civil Engineering Reference Vol. 2 169 Seat No CIVIL ENGINEER Licensure Examination ‘Thursday, May 2,2002 08:00 a.m. - 01:00 p.m. STRUCTURAL ENGINEERING AND CONSTRUCTION SETA INSTRUCTION: Select the comect ansver foreach ofthe following question ak ony one answer cach ie by Shang the bos corme-ponding i teletier ur choice on the answer sheet provided STRICTLY NO ERASURES ALLOWED. Use pencllno. 2only pues ctr hr niu PE coast 2 Wi antag tcc ie wnat in ne C30 hic he ol Dee 3. Which ofthe following gives the component AMD ee mC a the Y-axis in Newton, B 290 D. 390 Situation 2 - For the continuous beam shown in figure AN-51: ‘1 Which ofthe following gives the moment equation f =f 3M + 2M suspension cable is shown in Figure ME-76. the following most nearly gives the tension in segment BC in C 1854 Diaia7 8. Which of the following most nearly gives the angle fir degrees A 4218 C4978 B, 3568 D. 5241 9. Which of the following most nearly gives the total length of the cable in meters, A. 2948 C2547 B. 3225 2817 Structural Engineering ‘and Constraction 170 May 2002, 9-2 8 equal 0 the net width along bolts 12-4 10, Which of the following most nearly gives the value of bin millimeters. A286 C521 B 374 D.197 ‘1. Which of the following most nearly. gives the value of the net area for tension in plates in square millimeters. ‘A 3624 C3867 B ama D. 4178 a 12, Which of the following most nearly gives the value of P so that tte allowable tensile stress on net area wall not be exceeded, C539 kN D. A24kN Situation 5 ~The shear diagram of a loaded beam is shown in Figure ME-€3. 3. Which of the following gives the largest concentrated load (not including reaction) acting on the beam in KiloNewton. A 10 {ely B40 D. 20 |. Which of the following gives the maximum negative bending moment in KiloNewton-meter. A cs B30 Dio 15. Which of the following most nearly gives the location of a point of inflection fro ight end of the beam in meters ‘4. 287m C. 357m B 314m D.375m. Situation 6 - Section 5.11 of NSCP states the following for shear on beams subject 40 combined flexure and axial load: 54132 Shear strength, V;, may, be computed by the more detuled ‘aleulation of Sec. 5.11.3. through 6.11:3.2 531321 Formember tet to sharon fer only vex |( (rev, )e7| ea are but not ger han 03 {Fh greater than 1.0 in computing V. ‘occurring simultaneously with Vy, 6), where My is factored moment ‘considered. Reference Vol. 2 a7. ‘5113.22 For members subject to axial compression, Eg (11-6) may be used to compute V, with My substituted for M, and Vs d/2, not then limited to sown mented ga) ave eel epee pate seo fF a ite be expressed in MPa. When Muss computed by Eq, shall be computed by Eq, (118). ne 7 bad *ofed axial load normal to cross section occurring simulkancously bo taken as positive for compression, native fr fens elude effects oftension due fo creep and shimkage red momenta ection ei a sia lain patie ae eee oe So eae ee See oe ae cee nn arse 16. Which of the following most nearly gives the value of tke modified moment ‘Me, to be used at the critical section for beam shear, in kN-m. Avraz C614 B D. 865 ng most nearly gives the value of the shear force Vat 2 © 1561 a D, 1357 18. the following most nearly gives the shear force to be carried by forcement in kN ; . 180 D. 80 Situation 7 ~ Three altemative pipe systems are under consideration, Each has been designed to meet the project specifications. The installation, annial and hourly operating costs are summarized in the following Pipe system is to be used for 500 hrs per year and has a project life of 10 years, and then will become valueless. Use interest rate of 15%, Structural Engineering 172 Mayy 2002 ‘and Construction Pipegetmm > | 10am pipe | 150mm pipe Tnsalaion Cont | Paonomn00 | ¥2.20000000 “Annual = A Maintenance agente pean ope 80.00 1). Which of the following most nearly gives the capitalized cost of the 150-mm 1. p2656 45200 ©. B5.38065300 3. P310752400 , P3,654452.00 20, Which of the following most neatly gives the annel cos of the 10Dshm PIR parezisco C. Rseoas200 B. PASZA78.00 . 53540200 21. Which of the following most nearly gives pipe systems is the most Sonomical ‘A 10D pipe C. 250-mm pipe B Ast-mm pipe . Not enough data Situation 8 - Section 5.7.7 of NSCP states the following provisions for concrete eee Minimum cover, mum (@) Concrete cast against and permanently exposed to earn =-.75 {8} Concrete exposed to earth or wea 20 min through 36 mm bars... 50 (©) Concrete not expose to weather arin contact with ground = labs, wall, joss 3mm Beams, columas Primary reinforcement, ts, straps, sts folded place members 20 min basen smn, and footing in Figure CO-39. 22. Which of the following gives the minimum cover for Detail 0. A. 40 mm ¢. 20mm B. 15mm D. 50mm 23. Which of the following gives the minimum cover for Detail 02: A. 40mm C 50mm 5. 15mm D. 75mm 24, Which of the following gives the minimum cover for Detail 03: A 40 mm C. 50mm 8. 20mm D. 75mm network fvities, duration, and cost unde diagram is shown in the 27. For a total cost of P22,100, which of the following the job can be accelerated for most ecomomical result ‘A. 17 days B18 days. Situation 10 - For the riveted bracket shown in Figure rivets have diameter of 25 mm. The connec h.of the following: gives the critical path of the network, E CDEP D. BEF F in of the following most neaely gives the cost of the project if ts to be ed to tminimum duration. pe ives the number of days ie days DiS days 25, P= 250 KN. The can be analyzed composed of vertical ‘Sanda moment caring stress in each rivet de to the vertical force alone acting at the centroid. ‘A. 145.6 MPa 90. Which of the follows instanangons center 0) ©. 1273 MPa D, 186.4 MPa following most nearly gives the shearing stress in rivet D in © 95.6 D, 1347 ost nearly gives the stress i nivet B when the is 200 mm to the left of cg, in MegaPascals, eae Cour rs D. 3418 174 May 2002 Pee ete Civil Engineering oa F100 ram 300 mm 300 m9 f Column ret expose to estar eta 03 Footing Figure C0-39 Pe temn Figure ST-13 Structural Engineering ‘and Construction Figure ME-63 f 6m um am Figure ME-76 “+ END Civil Enginee Roferense VoLite 177 2002 Examination Solutions to May @) Situation 1 (1 to 3) X-component of P: P= P.cos 30° c P= 450 cos 30° = $89.71 kN X'-component of P: Pr =P cos 6° Py = 450 cos 60° = 225 KN ‘Y-component of P Pr=Psin 6” Py = 450 sin 60° = 389.71 kN Situation 2(4 to 6) ¥ oe EPA u=am Three-moment equation for points 0-A-B: Mo Lo + 2My(Lo+ Li) + Me La + ood -e . Bor 81 7 pean aac 0 ‘2M, +Ma+27=0 Eq.) ‘Thrco-moment equation for points A-B-C: ‘Mg La + 2Ms (La * La) * Mc lat wl. 129)" ‘Structural Engineering Civil Engineering 178 May 2002, ‘and Construction | Reference Vo. 2 179 MaG) + 2Ms@+4)+0+81+ 300-0 | Ase ooo 3Ma+14Mp+381=0 Eq.) | wind singe (0 709% Bi=90-8= 1908" ‘Subtract: Eq. (2) «2-Eq, (1) x3 (a + 28d + 762 =0 AtjointB = 6g + 3My+ 81 =0 | [BFe= 0) Tea ee a (Maa) Tacos fi = 20.23 cos 19085 (ag)= 19.12 KN [eFv=0) Me~-2724kN-m (Tag) = 16+ 20.25 sin 19.08° (va) = 22613 KN AD Situation 3 (7409) ose um 7m | Total length 4 ofcable: Pe a L=h+hetly L=60¢ + sec P+ 750095" e = 6 sec 49:78" + 11 soc 19.08" + 7 see 5° L=29.48 m [=M,~ 0} To. mn Situation 4 (10 to 12) 16(6) + 20(17) = Dv( b 30s, 12Dy = 218 > Eq) or ron ot cet pa ae Dy= 070020», Ea.) ae Substitute Dy of Fa. 2) to Ba (1) @ a SD y+ 1210700223) = 218 a5 Dye 19.186 KN Dy, 19186 1 ag eres tos = pee’ (©) Polygon of forces at 150 mm “aimee | Net width = Gross width F holes #22 (2334)2 — 2(20)(23.34) cos 55° ae Net width (1-2) = Net width (23-4) 180 May 2002 g-204+2)+ 9 f2+300b- 6300=0 Considering path 1-2-3-4 or 1-2-4 Net width = 400- 3(34+2)+ 80, (050-8) q (200) eo? (150-1971) width = 400 - 3(34 +2) + + Netwidth = 40-904 +2)+ gos + Se Net width = 32222 mm Considering path 1-2-9 one Net width = 400 - 3634 +2) + 4q10) * 4(100) Net width = 901.97 mm_ CCitical net Area = 901.97(12) = 3624 mm Net area = Net with x plate thickness Net area = 322.22(12) = 3866.6 mmt Pa Age Fe P= 3624 x [0.6(248)] = 539,251 N P=5392 KN Civil Engineering Reference Vol. 2 181 ne @ Situation 5 (13 t0 15) Largest concentrated load = 40 kN ‘Maximuim negative bending moment = 25 kN-m Point of inflection (Paint of zero moment) x, 15 2s 1B ae EC Distance from right end =25 + x2 = 375 m. 182 May 2002 —_—_——— @ Situation 6 (16 to 18) ‘Weight of beam = YancX Area = 23.5 x (03 x 05) = 3525 kN/m 104 = 1.4(8.525) = 4935 kN/m R= 1450 + 4.905(4.5] = 236.104 KN -M, = 236,104(0.43) - 4.935(0.437/2 = 101.07 KN-m N.= 100 KN Vj, = 236.107 - 4.835(0.43) = 233.98 KN = AL 2 15 og Pe “5d 300(430) Ma = Me Net 4(05) 04 3 My * 101.07 - 100! My ® 81.445 KN-m_ ae [ Levan afer] we { 7 +0009) SG 2) 7} (600)(430) V_= 135,750 N = 135.75 KN Vy Ver Ve = 288 297527 N 085 Vee Va~ Vem 275.27 ~ 135.75 = 139.52 KN Seen 183 {Situation 7 (19 to 21) Capitalized Cost of the 150-mm pipe: orcs OM, AC-SU RC = FC=P1,200,000.00 (OM = P88,000.00 + 360(500) = P268,000.00 268,000 , 1,200,000 015” @+o15)®=1 K=P3,380,683.7 = P1200,000+ Copied Cont 100m pipe kare OM , ROSY asta c= FC Foonan000 i= Pg6 a8 0+ 1000 = e75a0000 76am, sn ran = 2750 50 + Tas” 4015) KePasoauas Anna Cont AC™ Kin 85099449(015) nwa ost Ace Peas a+i"1 RC= FC P1,600,000.00 ‘OM = P280,000 00 + 280(600) = P420,000.00 K-=P1,600,000+ £20:000 ,, -1.600,000—0 015” @+0.5)"—1 K=4,925,355.33 ‘The system with smallest capitalized cost is the most economical, Thus the 150-mn pipe is ‘he most econamical, ED Situation 8 (22 t For Detail 02, minimum cover is 40 mim For Detail 03, minimum cover is 75 mm Structural Engineering 184 May 2002 ‘and Constraction Situation 9 (25 t0.27) Part 1 Network Diagram (Normal Conditions) =3 ‘The paths from Oto 4 are: Path D-F: Duration = $+ 58 days Path A-C-E-F: Duration=5 +7 +445 = 21 days Path B-E-F: Duration=5+4+5~14days Part 2 By accelerating A-C-E-F, the critical path would still be the same, Note: Activities Band D may not be accolerated since it not included to the new critical path eae 185 Ralivity A (Accelerated) B (Normal) D (Normal) C (Accelerated) E (Accelerated) F (Accelerated) 3 Gost for minimum Duration > ‘Duration ‘Minimum duration =3+5+343=14 days What if? Ifa new critical path is established Gay D-For B-E-F), ay to accelerate the activities along this new path, Part 3 Maximum available cost is P22,100: The normal P22,100, we P1,100.00, (fom Part 1) is P21,000. Since the maximum accelerate some activities with an additi Accelerate items along the critical path: Activity A: (Maximum crash time = 5 ~3 = 2days) Crash cost Normal cost Cost of rash” mal duration ~Accéerated duration Comat erasing = MO2=2800 «2000 sr day Activity C: (Maximum crash time =7 ~5 = days) tof eahing= 2580-340 «50.00 por ay Activity: (Maximum crash ine = 4-8 =1 day) Comofrting = 1500=100 — 50900 por ay Activity: (Vicinmum crash tine =5 3 = 2 days) Cost of crashing = 1800=1.000 409.00 per day. 5-3 186 May 2002 sertina Coneerscion oe 187 Reference Vol. 2 “The best aclvty ov activites © crash fst ave those that have the least Paes coat of cashing, Among these activities, the best to crash is Activity C while Activity Es the worse For a total of PLJ00, Activity C can he crashed for 2daysforacostof2x | 250 ~ P5000, and Activity A can be crashed for 2 days for a cost of 2 x F 00 = Peo. a0 wety + For most economical project duration, the project may be crashed for a Rae ey t total of 2+ 2=4 days. Bey 200 tg. 1125%108(200) a ‘Thus, the job can be accelerated to 21 - 4 =17 days = 10,000 a= 5487805 N =54.878 KN G Situation 10 (28 to 30) T: 112.5108 (350) Direct stress: - oo Ps «10° = Rains = ?/4=250/4 = 625 KN Sw" Saray} 5 aloo OPN = 25:10" 752 Pa a= $6057 405) Ra= (64878) (6254 960877 ~16777N ‘Stress in rivet D: ‘Stress = ‘= 341.77 MPa T= F250) = 250 x 10%(250) T= 62,500,000 N-aum Die + y) = (1802 + 2008) «4 Ra" Bat + 37) = 250,000 men? Row EY = £2-500,000(200) "Fer? 250,00 Rox™ 50,000 N = 50 KN > Tx _ 62,500,000(150) = = SOON) = 97,500 Tere) 750,000 i Roy=375KNY Ry Re Total reaction at D: 188 November 2002 —_—_“trmmryorstlontagietring eee Seat No! CIVIL ENGINEER Licensure Examination Saturday, November 23, 2002 (08:00 a.m. - 01:00 p.m. SETA INSTRUCTION: Select the correct answer for each of the following questions. Mark only one answer for each item by shading the box corresponding, to the leter of your choice onthe ansvger sheet provided. STRICTLY NO ERASURES ALLOWED. Use pencil no. 2 only. ‘MULTIPLE CHOICE 4. Bind the value of » in 3°5)(9*6) =3°(22) Note: The expression * means power. ABS 95 Bo Ds 2. Whatis the value of y in the harmonic progression 1, 1/5,1/9,1/x,..? A10 cn B12 DB 3, Xis 12% older that Y while Yis 11% older than Z. By how much, in percent, is X older than Z. A Baa c. m432 B, 2178 D. 2.55 14, P36,000 is to be divided among Arturo, Bemardo, and Caley such that their shes in the same order, form.an arithmetic progression, Bernardo's share fs three times that of Arturo’s, How much is Bernardo’s share? ‘A P6000. C. F18,000, B, 12,000 10,000 5. In jangle ABC, A = 116° 19', B = 55° 30, and C= 80° 37. of sides A155) B. 13.21" D. 11565 6 A clei circumscribed shout a hesagon Determine dhe are of the jhexagon if the area outside the hexagon but inside the ci eq. ‘A. 733 59. cm. C717 sq.cm. B. 724sq.cm m. ‘A Tateral edge of the frustum of a regular pyramid is 1.8 mong. The upper aso is @ square 1 mx 1 mand the lower base 2.4 m x 24 m square. Determine the volume of the frustum in cubie meters, ‘A. 46 Case B33 D. 65 8, A solid spherical stecl ball 20 cm in diameter is placed into a tall vertical Gylinder containing water, causing the water level to rise by 10 cm. What is the radius of the cylinder? A121 1028 B. 9.08 D. 1155 16. Ww. 18. 189 ‘one meter diameterat the top Gem high holds saltate rom the bottom. How many ci pee sait does it contain? ee bss Thai clone by the Guve s+ 25 revcled aboot ine Strcie chan eenal | ene ea aes can aba 5, 4509 Sittin vere ot A320 Rae nes vse a9), 8020, an 2, Wht te ai A 3/4 C.3/2 B. 5/4 D1 ithe dacs bron pins 4108 an 3.29404, ht i ha ee Be 53 Duties tara efideyiet nics’ epsisal a Sirdar a ey dathe derivate oy = 3°67 Ieee Eeeaett jose pote a © ¥6r-3in6) PG BL sbyine export nates main in cs ope pn CRU ate eReenIneS tn are ee "eas ees oes Bi tse Pires oe tee ad Sicpeiind ieee tear era RSE hohe eg ye yee ee tel Fea How lat teupprend moving down tn) s ene D, 058 th of one ae ofthe curve whose parame equations are and y= 2 2c B See A body starting from point A is given an acolertion which uniform! Increase from zero at Ato 18 m/s?at Bind seconds. The inital velocity at ‘Aism/a: Find the detance tuvelled at theend ofdsecondss Clean bam i190 November 2002 20, TW contractor can Fuy tracks for P6000 each, or Feat Gem for PL.200 ‘mack por day. ‘The truck has 4 salvage value at the end of i sofa le of 5 y 20,000 per truck Using the annua ermine the numer fdas por year that each trick must be used to warrant is purchase Use Sinking hand metho of deprecation AW 155 87 D. 198 What ithe diffence between the sums an anmsity de and an ordinary nnuity forthe following date: Periodic payment: P14,000 1éyears 104 compounded quarterly s Bieaite ig 2 ‘At what interest rate, compounded quarterly, will an investment double in five years? A 16.6% c 141% B 151% Dr 2 ‘The tangent of a simple curve from PLL to P.C. has a bearing of N 65° the other tangent from PIL. to PT has a bearing of N 55° We Ata ‘m from P.C. along the tangent through P.C,, the right angle offset fon the curveis 6.2m. Find the radius of the curve ‘A. 16575 m. C 17523 m B, 15472 m D.18176m With the use of an engineer's level, the reading on a rod 80 m away was found to be 2.82 m. ‘The bubble was levelled though § spaces on the level fube and the rod reading increased to 2884 m. What is the radius of curvature of the level tube if one space on the tube is 0.6 mm long? © 335m. earthwork on an Bam-widle 475 F150 C1498 me D, 12.615 m? following areas bounded by the waterlin | and 5, Each contour are at 2 m interval, 5150 mm? Ag= 4140 me 010 me ‘As= 3,150 m? of a lake and the 3 = 4,650 m2 Determine the volume of water in the lake ‘A. 35500 cu m. ©. 37,800 cu. m. B. 36,800 cu m. , 38,900 cu. m. cd curve. Bo uS1m from CtoD? A, 328m B. 378m 29, A pavement has a modulus of elasticity of 120 "MPa and a sti 05. Determine the modulus of elasticity of the subgrade. ‘A. 16MPa B17 MPa carve of equal radii connects two parallel ‘chord from P.C. to P.T. is 140 m. Detormine the ‘otal length of D. 1467 m_ 28, A student recorded the following number of paces alter v aking a distance (0f 50 m repeatedly as 71.5, 72.0, 70.0, and 695. He war,ted to measure the distance betwean two points C and D. He recorded 7 ff paces from C to B of back as 465, 468, 463, and 460. "What is the distance C. 15'MPa, D. 12 MPa 30. Cracks in concreié-pavement due to temperature jions can be prevented or minimized by: Res “a. insuring proper consistency and strength of concrete mixture proper preparation of subgrade . Continous pouring of concrete D, providing expansion joints and sectional pouring of concrete Sigs WD CB mB we ma we me fc WA BD BWA Wk WA MA BE BD mo BD MO Mathematics, Surveying, and "Transportation Engineering 192 November 2002, Solutions to November 2002 Examination gm =369=35 GNF 3aagsgae 3” 2e=17 x=85 Given the harmonic progression: 1,1/5, 1/9, fx cals 1, §, 9, x form an arithmetic progression with common By inspection, x= 13 X=12Y Eq () Y=1NZ > Eq. ‘Thus, Xs 24.32% older than Z. Let A, B, and C be the shares of Arturo, Bernardo, and Caloy, respectively. AtB+C=36000 > Eq B=2a Eq. Since their shares form an A. P. ‘Common difference, d= B-A=C-B 2BeA+C > Eq. (3) Substitute B in 2) to @): 2(04)=A+C C=3a 54.) Substitute B = 2A and C=3A to Eq. (1): ‘A+ 2A +34 = 36,000 ‘A= 6,000 B= 2A = P12,000 (Bernardo’s share) ms Given angles of spherical triangle: A = 116° 19, B= 55° 0, and C= 80°37" 193 ‘Ay Area of sector - Area of triangle 25-2) st asiner 360" 1=5.253¢m Aseagon™ 6 « Area of triangle Ascagar ™ 8 Annagon= 717 54. v= Hla, +4. +JaiAe] r= (242 =576 me? z= ()P=1 me Fe= 1.89 0.989952 A= 1508, v= M8 fren V=a5om Civil Engineering Reference Vol. 2 195 ee eee eee | ‘ta | Let (7) be the coordinate of the centroid. Then, 194 November 2002 Ge ‘Volume of ball = (Volume rise)ane fap nag) Absci F = ASC «DAD R=T155 cm | 3 | nti, y= 108746 « tees cs | ax ‘The distance tween pins Fy) and Pa, 2) en by AP + (ye = ya)? + Ge~ za)® a» | on ‘Reduce the equation to standard form, (<- K)?+ (y-K*+ (e-D2= 2 Sa ‘The curve 2+ y= pare oh aude 5 vith conterat (0,0) Fromthe second proposition of Pappus: V=Ax2nR Vx) x 2n(10) V= 49348 cu. units DY «Rendered sod (1) = 100,2(11)> ~ 45(11)2 + 264(11)] = 12,100 Mathemattes, Surveying, and Civil, ineering 196 November 2002 ‘Traneportation Engineering Remrenee Vol 2 When x= 4 Os x=-2sin@; y=2-2c0s0 Note: This curve is a cycloid where 2is the radius of rolling citcle and 0 is the angle of rotation (0"- 360° for one are) ‘The membership is largest (relative maximum) when x = 4 and thatis in 1978, } maa tmeten s+ ffl) aw . %, From the figure shown, de (2-205 0) 40 hens = 2 0 antmea By Pythagorean Theorem: = j po [eezenoay aes y= vx00? = |. rs d Ta 20s costo ce soo [EES cme aaa ayes sia OF Deas od lm soa” pao Restor D4 wm, btst cat 1 ae 4M narens oe [ren ‘Therefore, the upper end moves down at the rate of 1.79 cny/s ” but Ji=c0s0 = V2 sin® av 5S : [fierce sea ang ara Panta pees 30 sea[ acco)” =-a[cos ae Ze ee 00-0 s-16 ‘Note: The length of one are of CYCLOID with parametric equations <= o(0-sin6) and y = a(1 ~ cos) is 8a. x1= 0178 . Yeloey diagram 4 5 Area =4(66)- #(G6)= 168 m of the trac aa SEL AC = Annual interest on investment + Arimual operation nal cost atenance cost + Annual depreciation (using sinking fan) (EC-SVyi = (i+ OM+ ae at 3 "- nooo « 200 =100,000)018) t ‘AC (@00,000,0.14) + 20900 + AC = P237858.48 Let N be the required number of days per year, then ‘Cost of renting = Annual cost 1,200 N = 237 898.48 N= 198.24 hrs say 199 hours Ordinary Annuity: Secale 40025) — 4 000{(1 +0.025)" =I) _ po 159,665 0025 Civil Engineering Rewer a ae Tamiya jum ~ (Forainary nencty)(1 + 6 Sum = 15960505) = P2213657 Difference = 2,213,657 - 2,159,665 = PS3,992 Or: Difference = Dilference = 1mm +045)" —1 0035 (0.025) = P53,991.62 200 November 2002 os 670 180 aa APAV+ Ast Ast Ac A=W [4(L.8) +1467) + 144.75) +405) Scien yee er Vo 8 ty tera +24 Aa ve 2 [6,150 + 4(5,010 + 4,140) + 2(4,650) +3,150] = 96,800 cu. m. Mathematics, Surveying, and "Transportation Engineering 912.615 Reference Vol. 2 201 ——— $$$ Intiange () aap sino= 22 jo-asiry 29-9858 =1 Intriangle (é) 35 sing= 3 R 35 sin agi71* = 23; R= 408.33 m ei R= 40833 ARI. , n(408'30)(9.838°) tend RL apy 14017 m 180° ‘180° ny zs For the 50-m distance: Average paces TEEST2S7OO85 7075 paces 50 Pace factor = 20 = per pace Faas 707067 meter perp For points C and D: ‘Average pace = Bee = 464 paces Distance = Pace factor x Average pace = 0.7067(464) Distance = 327.91 m rane os m2» Stiffness Factor, k= 3| sere ose eso 20 E, ete as (0.5P= SR; Enngrde™ 15 MPa Cracks in concrete pavement due to temperature expansions can be Prevented or minimized by providing expansion joints and sectional ‘pouring of concrete. 202 November 2002 Mygraulics ang Geotechnical Engineering ‘Seat Now CIVIL ENGINEER Licensure Examination. Saturday, November 25, 2002, HYDRAULICS AND GEOTECHNICAL ENGINEERING 02.00 pan. - 06:00 p.m. SETA INSTRUCTION: Select the correct answer for each of the following questions. sm by shading the box corresponding to the letier of your choice onthe answer sheet provided srk ICTLY NO ERASURES ALLOWED. Use pencil no, 2 only. MULTIPLE CHOICE 1 ‘A 10-m long tank contains water ata depth of 2m. Compute the minimam ‘Pressure at the bottom of the tank if it accelerates to the right at 245 m/s, ‘Assume the sides of the tank are high enough to prevent spillage. Choose sto the correct answer. C. 7804Pa D. 7391 Pa flows into a horizontal channel as shown in Figure 02. Which jest to the flow through the gate per meter width ‘= 6m. Assume that the pressure distribution at i ction losses in the channel, 3. oo a Siuice gate in the form of an arc of ezele of radius 6 m is shown in Figure 04 Which is ofthe following is closest io the vertical component of the hydrostatic force acting on the gate ‘ASS KN pormeter . 24 KN per meter BL S2KN per meter D,2S8EN prin Anoaden oc having urn rosso wl ot ter with 20 cy rojecing above the water surface and i placed in oll ep. gr. ~ 085) wi Fioct with 1d.om progecting above the oll urface. Deteroine the total depth of the Block. Choose the one coves tothe correct answer A osim ¢ 017m 3 : 5 bam ‘A masonry dam (density = 2500 kg/m) i langlar in cross-section with AONE Baia aSl bask ide of me The Hight of wate behind the ce of the dam is 20:m. Determine the location (measured from the Civil Engineering Reference Vol. 2 203 ———— eee A 73m ne © 67m D.77m SITUATIONAL Situation 1 — The jets from a garden sprinkler are T-inch in diameter and are normal to the 2-feet radius as shown in Figure 07. The pressure a of the nozzle is 60 psi. Neglect the velocity n ane had at the base Use coefficient of velocity C= 0.80 and coe! sont of contraction C;~ 1.00. Which of the following is closest to the velocity of flow cf the jet in ft/sec. A936 C547 B. 755 D. 863 Which of the following is closest to the force exerted by.the jet on the C6 Dia the followiligris closest to the force F applied on each sprinkler from the center (as show) to malntain eiisam C190 D. 162 Situation 2 - In the figure shown, the 50-m pipe is 60 mm in diameter. The fiid flowing has mass density of 920 ke/m® and dynamic viscosity of 029 Pars ese ee ea pales pret eet nace peli A7Ami/nfomiw2 Cassn/telom ior iiee cect aes hte ete wae B. 039 D. 0.63 a, Which ofthe following most nearly approximates the Reynolds Number of 4, 351 c m2 BSL D. 135 tre Power generating system is shown in Figure 13. an voir to a lower one passing through a per second. The total length of pipe Wm. ‘The pipe ciameter is 250 mat the 120. “The water suriace elevations of 50 m, respectively. early gives the velocity of low in the pipe in Whi of cath Whi ofthe folowing most aly giver he fiona hon st in the pipe a3 aus a7 D271 Which of the following most neatly gives the power generat ‘bine llowatts? Neglect minor las," PWS" Benerted by the Hydraulics and 204 November 2002 Geotechnical Engineering xi ci B 270 D150 Situation 4 - The pump shown in Figure 16 draws water from a reservoir and discharges it into a nozzle at D. The length of pipe from the reservoir to the pump is 150 mand from the pump to the nozzle is 1500 m. ‘The diameters before and after the pump are 450 mm and 600 mm, respe 5 both pipes. Z)= 4m. The pump is to operate such that the discharge will be the maximum possible. 16. Which of the following most nearly gives the velocity of flow in pipe AB? ‘The atmospheric pressure is 95 KPa absolute and the vapor pressure KPa. Use f= 002 B elmys Dis 17, Which defollowing most nearly gives the eloiy of flow in pipe 2D? ‘A 2iave eismys” is 3 igu/s D, ia2m/s 18, Which of th following most nearly gives the misdmum theoretical rate at y be pumped trom Mie reservoir? vity of 269. After oven drying, the mass of the wing most nearly gives the density of the in-situ soil? ©. 131 Ib/fe D. 113.4 1b/46 20, Which of the following most nearly gives the porosity of the in-situ soil? A051 cox B, 042 D032 21, Which of the following most nearly gives the degree of saturation of the in- situ soil? A 75% ca B 45% DL 35% D. st 23, Which ofthe following most nearly gives the angle ofthe failure plane? Ao C. 38° B52" D. ase 24. Which ofthe following most nearly gives the maximum principal stress? A, 10,000 pst C9400 pet B. 14,000 pet D. 18,500 pet 205 ' pier is shown in Figure 25. The the rigid pile cap, The piles the depth Hs = 3 ft below the botiom of sea, ‘om the sea bead to the bottom ile cap is 17 feet son pile D? foil st nearh eh sh 2. Which ofthe following most neasly gives the minimum moment in ple D ssuming point of contraflexure at depth of 10 feet bow the pile cap ae Cane B. 30 kip-ft D. 50 kip-ft hac ein Maat een ler meee aN aoe adepthof D-~8 fet Allowable cetementis 2 inches, Ue following most nesily gives the overburden pressure atthe ‘het foundation tone pertguaeoot 3s 28, Assuming a comecton factor Cy ~ 042, wich ofthe fl gives the allowable Bearing capaciy of the matin tors yer square foot? A208 C298 B. 243 D. 3.66 30. Which ofthe following most nearly gives the factor of safety against bearing capacity failure? Lee ya A 2d can B24 D.21 Figure 02 208 November 2002 m2 Solutions to November 2002 Examination = 21.209 h=0751m Minignum pressure: poh p= 88100751) p=73673 Pa ha Cxy = 085 x 1= 085m Energy equation betes © and © neglecting lo Consider 1m ength of gate [= 2a (Gx Do = (0.85 Tor m= O14170 141702)? = 101.043, (097993 v2= 101.043; v= 10.154 m/s Actual velocity at ©, = Cove = 0.95(10-15) = 9.6167 m/s Discharge = Ao v= (0.85 x 1)9,6467) = 8.2 mi/s per meter Mydrauties and Geotechnical Engineering Civil Engineering Reference Vol. 2 209 eee et as Giver Rectangular channel with b= 2d (most efficient) Q=2m/s n=002 p= 13m/s An Qa 2 215385 me 213 A=bd= Qid= 20 = Hydraulic radi owl ens 13= 1 ossaspe sve a2 E s= 0.00208, 85, d= 0.877 m 4/2 = 0.877/2= 0.4385 m Considering -m Jength of gate Fy=y(Asx1) Fy=981(6.261%1) Fy=32 kN (per meter) as 1 water not Fora solid homogeneous body floating in a homogeneous liquid: Sinty Shoat Vaepicd = Vinay Myarauttes and Geotechnical Engineering 210 November 2002 Let Sy be the sp. gr. of the wood: Tawehe aqt-29)= 5 (ant SH=H-20™ >F4.0) oli Agi-14)= 58 (cH) a5 S)H=085H-119 > Eq. (2) [Ss H= SH) H-20=085H-119 H=54cm=054m as ‘Consider 1 m length of dam: Rm Ryn = RM-OM Ry xy = WG) ~ Ff) Ry-W y= 2500{4 12} Ry=360000kg = 1 4 Fy=yRA Fix = 1000(3 )@0* 1) Fic = 200,000 kg, 360,000 = 360,000) - 200,000( 3.) Dua T= x47 4.29 m a! eo -n=7704m Q Situation 1 (7109) Ye (en in A Energy equation between O and A neglecting losses: [Eo= Es} vat, Pa Civil Engineering Reference Vol. 2 211 —_—— ee ee repaint aria te U4 = 94.83 ft/s (theoretical velocity) Actual velocity, v= C, 4 ~ 0.80(94.43) = 75.54 fsee Q=CAn%4= (Cx C) Anon Q= (108) ¢ Gh)" 443) = 0412 80/5 Dynamic Force, Fp = 2t » g = 04121624) 322 FQ) = Fo) = (6031)(2) = 120.62 Ibs Dynamic Force, Fo (7554) = 60.31 Ibs [2Mo=0} i Situation 2 (10 t0 12) Lam ‘Taking level 2s the datum: Energy E, = 12m nor £ =D = 200.000 Energy, E:=0+ 2 = Pw 200,000. 99 165 y pg 920x981 ie Since E> Fy the flow is from 2to 1. Energy equation between 2 and 1: Ey-lym Ey 22.16 ~ly= 12 fy=1016m ‘Hydraulics and Civil Engineeri: 212 November 2002 cobelemeitnrete Reference Vol.2 213 ppm COSI Soe LRN AER “Assuming laminar flow (R: < 2000) ©) Situation 4 (16 to 18) . tang | ‘apgD" Do 0.711(0.06)(920) u 029 Reynolds Number, Re = 135-4 <2000 (laminar flow, OK) Since the pump is above the water surface of the source tank, the pressure at the inlet (at 8) is always negative 7 ° As the discharge increases, the pressure at B drops, To avoid cavitation, the absolute pressure at B must not fall below the given vapor pressure of Since our assumption is correct, then | | 35 kPa, Discharge, Q = 0.00201 m'/s x 3600 s/he 7.24 mY/hr (ftom 2 0 1) Velocity of flow, v = 0.711 m/s Reynolds Number = 135.4 Situation 3 (13 to 15) Eners iation between A and B: $y equation between, (using absolute pressure and datum at) Eq —ypo= Ea sg EBs 2g ¥ Dr nee 5 PRD) os UE ODS 381 045 2g 2g 9a1 1» 3.692:nys (velocity of flow in pipe AB) ‘Theoretical discharge, Q = Av = § (0.45)? (3,692) = 0.59 mijs ‘Velocity of flaw in pipe CD: ‘Aco Yeo = Aas tas Frictional head lost y= FOS} co = + (045) 6.692) Frictional hea lost y= 389m co = 2.08 m/s Situation 5 (9 1021 Dn opine at Om 9 ee 0+0+197-389-HE=0+0+50 We=487lbs G=2.69 HE= 111m é Density of the in-situ soil, y= ~ = 567 — aig ipyee Power, P= Qy HE=0.15(9681)(143.11) = 210.59 kW v~05 Hydraulics and 214 November 2002, Geotechnical Engineering We WAW, _ 567-487 _ Moistarecontent MC= 72. WM = SST AT — axes Fea ge NY e= 07234 ’ [ome= se} 2.69(0.1648) = 5 (0.7234) 5= 0611 = 61.1% (degree of saturation) Porosity, n= —£— = 07254 — 9.4198 Tee T+O7md @ Situation 6 (22 t0 24) a a= (4.2007 +(6.300)° =7,571.66 Tingle In triangle 2: = 90° -§~ 90° ~ 33.69" = 5631" a e Reatand = 7,571.66 tan 33.69" = 5,047.76 C= usec) = 7571.66 sec 33.69" =9100 Z : ‘Wongle2 Angle of failure plane, 6 oo a+ 29= 187 563° +20~ 1807; 0-61.88 Civil Engineering Reference Vol. 2 215 El Situation 7 (25 6027) bro) mea at 3 fe H=S kips — (shear force in each pile) A eee! rao. Mp= Hix 10 Lye ‘Mo =5(10)= 50 kip-tt ‘T= 20(20) = 200 kip-ft r=18ft Hydrauties and 216 November 2002 Geotechnical Engineering, Tea = 20, 2006) ex BO, 00 sxerpe Paw 200. 20006) _ 8.03 dips gages rae BH, BOC _ 454i ae aes <0 Pa+ Pa+Pc+ Po-200=0 45 + 48.33 + 51.67 + $5 ~ 200= 0 (OK) @D Situation $ (28 to 30) 5200 tons Overburcen pressure = pressure of the soil romoved to place Overburden pressure = y Dy= 120(8) = 950 Ib/ft (1 ton/2000 Overburden pressure = 0.48 tonsyft? ‘Bearing Capacity from standard Penetration Test (SPT) ‘Allowable bearing capacity: = O41 News pu (KP8) peo ake re ne rece eee w= (28) en nan y= O77 logis (218), ey s2. om > 24 KPa (Peck, 1974) 8) 'm = effective overburden pressure, KPa i if the groundwater level is within a depth B f the footing, 217 the depth of the groundwater level if beyond 3 from: pier groundwater leval if beyond B from the pe= allowable settlement in mm groundwater table was not spe 11.16 ened a= 0A1(11.16)(60.) = 252.44 kPa x M47 PL 101325 kPa y= 98.72 poi x 144 = 4855.96 psf = 4,855.96 psf = 2,000 = 2.43 tons/sq. ft. =93.72psi Factor of safety: Fs.-—t =~, 4 = maximum applied foundation stress = 2200 T0c(32) 25 tons/ sq.ft. 20(8) = 960 psf = 2000 = 0.48 tons/sq, ft #5,- 28 api 25-008 octoral Eogincering 218 November 2002, Serncsya Conetrnclon SSS (CIVIL ENGINEER Licensure Examination Sunday, November 24, 2002 (06:00 a.m. - 01:00 p.m. ‘lect the correct answer for each of the following questions. ea em by shading the box corresponding to the leter “Mark only one answer for Of your choice on the anewer sheet provided. STRICTLY NO ERASURES ALLOWED. Use pencil no, 2 only MULTIPLE CHOICE tution 1 ~The 300-Newton block shown in Figure 1. at Steal orizontal pane efor the force Fis applied at #= 0 ee Pp aNeortone an is neo a sllowing most nearly gives the time, in seconds, when the me © 176 D321 2. Which of the following most nearly gives the velocity of the block, in meters ‘after five seconds? fl Bibs 5. Whit the folowing most nearly gives the distance tmelled by the Thee ia mete after ve seconds? c 185 llowing most nearly ifthe concentrated load D. 098 5. Which of the following most nearly gives the deflection at the free end in rillimeters, the moment were acting alone? B18 182 ree Boa D338 , 6. Which flowing mentee gv the meme flo nna a3 D.A56 219 towing best describes the Casati of tructure (3). indeterminate to the second degree determinate terminate to the first degree ‘ng best describes the classification of structare (b) ‘A. Statically indeterminate to the first degree 1B. Statically indeterminate to the second degree C Unstable D. Statically determinate 9. Which of the following best describes the classification of structue (:), ‘A. Siatically indeterminate to the first degree B, Statically indeterminate to the second degree © Unstable D. Statically determinate Situation 4 - The activities, duration, and cost under normal and accelerated conditions fora network diagram are shown in the accompanying table. ©. Activity 4 2 ithe budge s P22600.09, wih ofthe fl aximuim budget is P2260.0, svhch of the following most neat gives the reduction in the number of days to complete the proce ”: ad Ee a? Dé Situation 5 ~ The continuous reinforced conctete beam shoven in Figure 05 ig Subjected to a uniform service den load of 16 N/m aad «series ee bad of 32 KN/m, ruling in the bending moment diagram shown, Tents percent of the live load wil be sustained in nate walle 80% wil Re Applied only intermittently. The concrete stength f= 172 MPa The Modulus of elastcty of concrete is given bythe expression Be A700 Sart) and the modulus of raptare given by ter epresion f= Oo 220 November 2002 595.26 Unless stiffness vahies are obiained by @ more comprehensive with the effective Fq.97 ‘Mg = cracking moment = vs ‘My= maximum moment at siage deflect J, moment of inertia of gross concre neglecting reinforcement ‘moment of inertia of cracked section transformed to concrete distance from centroidal axis of gross. sectiory neglecting reinforcement, to extreme fiber in tension 595.27 For continuous members, effective moment of inertia may be taken ‘as the average of the values obtained from Eq. %7 for the critical positive and negative moment sections. 5.95.28 Unless values are obtained by a more comprehensive analysis, additional long term deflection resulting from creep and shrinkage of flexural members shall be determined by multiplying the immediate deflection caused by the sustained load considered, by the factor SEE Toy ee p'= reinforcement ratio for non-prestressed compression reinforcement fat midspan for simple and continuous spans, and at support for Eq, @-10) stained Toadls to be equal to 5 years or more. 20 ‘months, 14 6 months... 12 3 months. are, Which ofthe following most neatly gives the effective moment of inertia at the supports (maximum ne ‘x Oors5 B. 0.00686 m*4 Which of the following most nearly gives the effective moment of inertia for the continuous member. ‘A, 0,00686 m4 B. 0.00879 m4 ©, 0.00815 m4 B, 0.00798 m4 221 ‘combined service dead and live wst nearly gives the additional feflection) after 5 years, under the C 612mm D. 735 mm. ‘sustained loading, A 467mm. B 840mm Situation 6 ~ As shown in the Figure 06, a secondary member inclined at an angle 8 =30" is composed of two plates each 250 mm wide and 50 mm thick. The secondary member is joined by bolts to the can be assumed that 1992 tual Code ofthe Pipes provides hal theellae ia connection at an angle of load Oto the gra bet cemaputed from the Hankinson Formulae ences O16 Where P is tho allowable load parallel to the grin and @ i the allow Teed perpendicular othe gai. - Serer ich of the following most nearly gives the minimum requred capac one bolt? x Ae Sea aca te asin D. 6S kN re wil occur inthe secondary member, which ofthe ollowing mest nearly gives the minimum diameter of the bolts? If failure will occur in the main member, gives the minimum diameter of the bolts wide and 75 mm thick are joined shown in Figure 7. The diameter of Which of he fllowing moet neany Ses et roof the tne member 28000 Cis s00'ma iain oy be 0 me Which ofthe flowing most nearly gives the mesum Load allowable erie stress onthe member isnot ecaodad. nh atthe ‘Structural Engineering 222 November 2002 ‘and Constroction Dl, Which of the following most nearly gives the maximum load such that the allowable Joad on the bolis is not exceeded. ‘AL 284 KN C67 kN B. 155kN D. 390kN, Situation 8 ~ A bill of quantities is to be prepare for the square isolated reinforced ‘coveveteeolums and footing shown in Figure OK he swell factor forthe backlill is 120%. 22. Which of the following mest nearly gives the quantity of excavation, in cubic meters: A 0218 C0465 B 0342 D.0570 23, Which of the following most nearly gives the quantity of concrete, in cubic meters: ‘A. 0568 0.200 B. 0372 0415 24 Which of the following most nearly gives the quantity of formwork, in square meters! oe ‘A302 asst B 256 D. 580 Situation 9 - A rectangular column is shown in Figure 09. It is acted upon by the following loads: “Axial load due to dead logd = 450 KN “The column is to be analyzed and designed using the strength method. Use b= 400 mm, k= 600 mm, and y = 0.75. 25, Which of the following most nearly gives the eccentricity of the column, in ‘A. 300 a3 B28 D1 26, Which of the following most nearly give the area ofthe minianum required steel reinforcement, it mulimeters: ie) “A. 5250 c. 3820 B 670 D. 4100 27. Which of the following most nearly gives the minimum diameter of the reinforcement, in alimeters. AaB C25 B36 D.32 10.005, 50010, ret 03906 Figure 0 Civil Engi Reference Vola 227 TABLE OL: Allowable Loads (KN) on One Bott (Double Sheas) Normal Duration mbm 202 10a ed rose Section racked Section rsca0rism 12000575 y= 310 mm y=1591mm ‘AT SUPPORTS: een na wile ress Secon ‘Cache Section w | 2 See = 0.00573 2g ‘yo 194 me y= 107 mn B AT MIDSPAN 228 November 2002 “w"Gnaconmmcion | Roterense Vol 2” 229 ‘TABLE G2: Working Streses for Visually Stess-Graded Unseasoned Structural timber of Philippine Woods | situation (1103) Before the block starts to slide, w= j= 04 When the block moves, =n 02 ‘f= 02N= 0.2600 + 408) f= 60+ 8 (Fu = 0] REF + f= P cos30" i END Far’ (60!) cos 30° (60+ 81) Beton “ANSWERS: af as 48 65 A BA 9=1.00185 #-19621-0.669= Zug rAd pet vcrrs i Rial BURG | Abi aC 2D MA BA s= [cose 1.962 ¢- 0669) at 5D 8 6A DB a ‘and Construction 230 November 2002 s=ox1e-o9s1-066s: |’ $ = 0334(53 - 2.252%) - 0.981(62- 2.2524 - 0.663(5 - 2.252) 5=1656m 1D Situation 2 (4 to 6) y a 233m =) 20/61 Conjugate beam on concerted les 10 Cconlgate beam on moment Deflection due to concentrated load, 5; = Mi’ 465kN-m! 8, = [4@)(20/EN] x239= = pe AO ms ‘200 000)100 000,000) Deflection due to moment, 8: = Ma’ 675kN—m B= 18(5/ED] x15 = ST _ 675,100)" ~ 7200,000(100,000,000) i =3375 mm Maximum deflection, 8 = 1 + &= 2.334 3975 = 5.705 mm 231 Avroller support has one reaction ‘A hinge support has two reactions An internal hinge gives one equation (zero moment) Figure (a) ‘Number of reactions:3+1+1=5 Number of equation =3 ‘has, the structure is indeterminate to the (5 - 3 = 2) second degree, Figure () ‘Number of reactions = 3+3 = 6 Number of equations = 3 + 2=5 ‘Thus, the structure is indeterminate to the (6~ 5 = 1) first degree. Figure 0) Numbor of reactions = 2+ 14+ 2=5 Number of equation 3+ 2=5 Thus, the structure is statically determinate © Situation 4 (20 to 12) Part 3 Network Diagram (Normal Conditions) bes ‘The paths from 0 to 4 are: Path D-F: Duration =3+5= 8 days Path A-C-E-F: Duration =5 +7 +4+5=21 days Path B-E-F: Duration =5+4+5 = 14 days ‘Thus, the critical path is Path A-C-E-E ‘Normal Cost = 1,000 43,000 + 12,000 + 3,000 + 1,000 + 1,000 ‘Normal Cost = 21,000 Seroctural Engineering 232 November 2002 ‘and Construction Pats2&3 Maximum available cost is P22,600 ‘The normal cost (from Part 1) is P21,000. Since the maximum costis to be 22,600, we celerate some activities with an additional cost of 1,600.00. to ‘Accelerate items along the critical path: ‘Activity A: (Maximum crash time = (Maximum crash time = 7- 5 = 2 days) per 300-802 ase 75 OO per day: tine = 4-8 =e ont of ting 15120 ery poy Act F heim cnhtine = 3-8 ~2de) Cost of crashing = 200=2:000 _ pyon00 per day 3-3 For most economical project duration, the project may be crashed for a total of 2424+1= 5 days. ‘Thus, the reduction in the number of days to complete the project is 5 days. © Situation 5 (13 t0 15) At supports Civil Engineering Reference Vol. 2 233 7 Ma= ihe ve f=07 JF =07¥i72 ~2900MPa = 0.00715 m¢ (as giver) 2.903(0.00715x 1000") Mo= * 66,956,290 N-mm = €6.956 kN-m 70 fe = 00057 ne i ($28) cams + [r-(2) a aa pee : (Gec.595.27 ) Atmaximum negative moment (at support) 1.= 000578 mé Solving for {at maximum positive moment (at midspan) 2.903(0.0138% 1000") Mo = 20198 (= 94,040,845 N-mm pees cat 94.04) (2247 ann (2 ] one oe 2.00578 + 0.00793 2 Additional long term deflection = lang term deflection x 2. é +505) Y = 0 (since there is ne compression reinforcement at midspan) (after 5 years) a= =2 +0 hi = 0.00686 ms he Solving for the long term deflection under sustained loading: Instantaneous deflection = 5 mm (given) Instantaneous loading = 16 KN/m + 32KN/m = 48 kN/1m_ Structural Engineering ‘and Construction 234 November 2002 Sustained loading = 16 + 20%(32)=224 KN/m ‘Since deflection is directly proportional to the load: Soneterm _ 5 ‘Additional long term deflection = 2.333 x 2~ 4.67 mm. © Situation 6 (16 10 18) ‘Two soem tk Seoondany rember Minimum required For failure in the secondary member: Length of bolt in member, use 50 x= 100 mim {Load in each bot (paral to grain) = 44/6733 kN Coad perpendicular to grain = 0 6 From the table, the minimum diameter is 16mm Length of bolt in member = 100 mm Load in each bolt at an angle to grain = 44/6=7.33 KN Angle to the grain, @=30" Civil Engineering Reference Vol. 2 235 Boer oe Pain? 04 Qc0s" 0 PxQ x Psin® 30°+ Qos" 30" ‘Try 13.nm bolt with P= 588 and Q= 4.07 5,584.07. ene . Sitsin® 90°+4.07c057a KN <7.35 KN tot OK) ‘Try 16-mm bolt with P= 847 and Q= 5.41 B47 «5a be 7 > ‘8.47 sin? 30°+ 5.41. cos* 30° BN Tek Oe) Minimum diameter = 16 mm D2 Situation 7 (19 to 21) Hole, 27 mm 9p) Part 1 Net area = Net width x Thickness ‘Net area = (400 - 27:3)(75) = 23,925 mm? Part 2: P= Allow tension parallel to grain x Net area Allowable tension parallel to grain = 24.5 MPa (Table 02) P= 24.5(23,925) = 586,163 N = 586,163 kN structural Engineering 236 November 2002 ned Constrection Pan's (Yakal belongs to Speces Group Bolt diameter=25 mm This isa two-member joint (single shea) of equal thickness. For this condition, one-half the tabulated load for a pce twice the thickness of ‘one of the members shall apply. Length of bolt= 2x75 = 150 min n parallel to grain = (946) = 173 P=173x9= 155.7 KN Situation 8 (22 to 24) Quantity of concrete: Footing = 0.6( Quantity of formwork: Footing ~ none Column = (0.15x2 + 0.2«2)(0.6 + 3:05) = 2.56 m2 Total quantity = 2.56? Situation 9 (25 t027) (450) + 1.7500) = 1480 KN 632.28 Eccentricity, ¢= 03 m= 300mm i 42, , 07(2114.28%1000) 000400) = 617 MPa From the interaction diagram, py = 0.017 ao eri 0.017(600 x 400) Civil Engineering ‘Seat Noz CIVIL ENGINEER Licensure Examination Wednesday, May 14, 2003. (08:00 a.m.~ 61:00 pan, MATHEMATICS, SURI & TRANS. ENG'G. SETA INSTRUCTION: Select the 4 answer for each of the following questions iy shading the box commesponing oth eer, of your choke on the answer sheet prov STRICTLY NO ERASURES ALLOWED, Use pencil no. 2 only. ‘MULTIPLE CHOICE 1. If log, 12=2.262, find the value of log, 3 A. 075 c Co5 B. 0.25 D. 10 2. Whats the sum of the numerical cocffcient of the expansion of A 64 C7 B65 D. 74 3. aand bare positive numbers. 4, -4,... form a geometric progression. 14, a, 6... form anarithmetic progression. Find the value of AG ra B.9 D7 4, Find the value of rin the equation (x + yi) (1-2:) =7 4 Al c. B38 D. 5. The sides of a triangle are 18 cm, 24 cm and 34 cm, respectively. Find the Jengthof the median tothe 2Lem side, in.cm 4 . 34 2 ae ' Dm 6. Accircle having an area of 224 sq.m. is inscribed in an octagon. Find the area of the octagon. ar ©. 23635q. m. D. 246.7 6q. m. 7. ind the radius of the sphorical wedge whose volume is 12 cu, m. with a central angle of 1.8 radians, A. 236m © 252m, B 273m D. 215m. 8, How far from a vertex is the opposite face of a tetrahedron if an edge is 50 cmlong? ‘A. 38618 cm, C. 39.421 om B. 40.825 cm D. 41214em 9 A truncated prism has a horizontal triangular base ABC, AB = 10 cm, BC = ‘em and CA = 8 cm. The vertical edges through 4, B, and Care 20 em, 12 cm, and 18 cm long respectively. Determine the volume of the prism, in A. 661 C685 B. 559 D. 574 Mathematics, Surveying, and x mee Civil Engineering 40 sing 3008 _"szemiaameme | semen 24 10, By us Pappus Theorem, determine the volume generated by revolvir ———— the aren in theft and second guadrans bounded y the Slee bee 23 A structure employed to direct the low of current in rivers witha view fo oe eee ti ctablshment moe favorable and Sued charvel an o yeven Sr A. 8563 ©. 9535 ‘A. dikes nrying away of river bunks Boa bar mole Le Tg ae fa ae reo tees tide slope of 2, a roud width 9-H tnd x ror secinal ofthe tise + Fog tsa =0 ia 31.7.5q.m, find the value of z in the following cross-section notar en B. 0.62 D. 058 o 12. Whats the radi ofthe cic 22+ y+ r= 10y-92= 0? ar eu BB Dia : D, #264 1B, Points C,7,2) and D (41, 6) are728 cm apart. Find the value of car diver ttvelling ata speed of 65 “agi Risen Gene ae ee 7am dunn he potpton escent Wie eee B dan D. tem erception, idenbiicaion, emotion and volton) tne seen? 114, Whatis the total length ofthe curver=4 sin? a2 ae we Cae Bor Dae 15. Whats the derivative with respect tox of cos (2 A otsings 2) € 2ane Beam @ee) , -2aing 16, Whatisthe curvature ofthe curve yé~ x atthe point (4,87 eee ues ‘The flloring interior angles ofa triangular traverse were measured with S04 oar the same precision. What the most probable value of angle tree v. iO to n/4 find the integral o sin 2d: ‘Angle Value (Degies) Novo Mourners 7 0302, : a : a 5. oe F ‘ 418, Cost of machine = 140,000 pesos; useful life = 8 years; salvage value = 10, 3 bt peson Determine heh yar deprecation wing the doubdedectning balance method, in pesos. A176 16182 ‘What isthe central angle in degrees of the curve whose radius i 200 m anc D.1g64 the distance of the midpointofthecuvetotheP Lis dm ne Pé0, 000 payable in 60 days or P57, 500 payable in 15 days, as cae lent annual rate of simple interest if paid in 60 days. D. aa ‘The horizontal axis of a transit was inclined at 4’ with the horizontal due to © 27% D. 329% non-adjustment. The fist sight had a vertical angle we next had -30? 20, Jue, in pesos, of an annuity of 20,000 pesos payable Determine the ero in the eaned meee ee , with the first payment atthe end of 10 years, if money A 746" C8 46" CC. Ps3.325 Di. PB1a24 1000 kg runs at 60 kph around an unbanked circular curve with a racius of 100 m, What force of friction on the tres should there be f© provent the car from sliding? ‘A. 2459 © 2778N B. 2671N D, 2568N 22 A compound curve hs, a common tangent 520 m long. The fist curve thuough the P.C. is a 3degree curve with a central angle of 50” Fine te sco fe one ctf i cred agi ‘A, 10843 37 B. 12652 D. 13043 242 May 2008 Mate sponation Rnginstring 22 key ee ‘Solutions to May 2003 Examination log, 12= 2262 P42 12, 2-3 Togs 3=logs3=1 ‘To get the sum of the coefficients, set all variables to one 1). ‘Sum of coefficients of (2 +B) = (1 +1)°= 64 Ifa,-4,8...formaGPy Cotta ae st ab=16 = > Eq. (1) If14,5/b,..foran AD; ‘Common difference = «-14=b-2 b=m-14 Eq.) Note: Two complex numbers are equal if their corresponding real and imaginary parts are equal. 12.0+hi=ctdiifa~cand b= 4 (e+ 2y)-@r- Then; r+ 2y=7 denynd ‘Add (1)+ 22 x +2y=7 Civil Engineering Reference Vol. 2 as Solving for @ in triangle ABC: 182= 342 + 24° - 2(84)(24) cos 8 8=3037° Solving for min triangle ADB: m= 3424 127 = 2(84)(12) cos 30.37" 8 =360/8 ~ 45° cos 45°/2)=8444/R | R= 93997 Aaugon "8% Aang ‘Avcapon = 8 (0.1397) sin 45° ‘Ateupan = 236.27 8 ‘The volume of opherical wedge of central proportional to the Yekame of sphere, hich & « 3p rae 3 Verte $97? 2 2? aang act Phe directly cal wedge of 244 Way 2003 De Volume = Asx 24 2412418 Volume = 39.686. 3 = 661.48 Ellipse: dx? + 25y¢= 100 eee? ee ae eee The solid formed is where c= b= 2 I vetune= Sten $a 3 Volume = $46)22= 88776 uns Nena oe Ppp Thema motel i Be rion 9x2 16y2-144 = 0 9:24 162= 144 Divide both sides by 144 r=asinO and r= 0s @ are cixcles with diameter 1 For the given curve, a= 4= diameter, Total length (perimeter) = xD = x(d) = dn 6x2 sin (2 +23) 246 May 2003 Mathematics, Surveying, and ‘Transportation Engineering is ‘The curvature of the function y = x) at any point i given by: ete z ee {8 ay=16 Y= 7 @@9;y=5R1 a», Sy = 81) _ 0 Ob 5 rage 0125 Curvature, k= = 0.002 near? ars ao when r=0;0=0 when x n/4;0~ x/2 Dex = fetoae oy [ie ogee OOM oe w fe 0m 4X aya) 2 0245 apna, av.-re{s-2)" sv aye tnao(1-2)'=Pars 1v-1a(1-2) ~4 ame ofeach ems bse Error, E= etano LY NO ERASURES ALLOWED, Use poncil no, only Where: : SITUATIONAL 2 angle of inclination ofthe telescope axis, ‘eronds ae Station 1A lock of weed O60. 040m ters in dimension wat thaw 8 = observed vertical angle of the object : into the water and floats will 0.18 m projecting above the water sisiace error in horizontal angle in minutes o ston The same block was thrown into a container of aliquid having a specie 4 ee nl ee avity f 090 and it floats with 0.14 m projecting above the surfs, 1. Betermine the value off 7m ©. 054m When two observations are made, the total error may be expressed as: ees at cea 2 2 Determine the specific gravity of the block Error = «(an ~tan 83) efrmine the specifi ra ok sep Bi 05a D.O751 3. Determine the weight ofthe block. A 1272 KN © 142i B 1549 IN Di 0951 EN [foam Pans ation 2 pro hana hata ftom wit of ane signs Sinton 2 pra] cholo wd of 6m and ae slope of2 horizontal to 1 vertical. If the depth of flow is 1.2 m and the ‘low ts 2040 m/sec 4. Compute the specific energy. ‘A al mn i © 179m B 254m Diam 5. Compute the slope ofthe channel ifn = 0014 ‘3 0.00005 © 00078 B. onooi7 D, 0000s * 6. Compute the average shearing stress a the boundary. A 69Pa coy Pe B55Pa D.82Pa Situation 3 - From Figure 0O1, itis shown that the gate is 1.0 m wide and is hinged ‘ at the bottom of the gate, 7, Compute the hydrostatic force in KN acting on the gate A760 cian . B 1530 ‘ 8. Compute the location of the conte of pressure ofthe gate from the hinge A 0887 m ©0517 m B. 078m D. 0333 m Hydraulics ana | Civil Engineering 253 252 May 2003 Geotechnical Engineering | Reference Vol. 2 9. Determine the minimum volume of concrete (unit weight = 23.6 kN/m®) A. GW (well graded gravel) C. GP (poorly graded gravel) ‘eed to keep the gute in closed poattion. y d B. SP (poorly graced sand) D, Str (wel pected sofa) ‘A Olozme C0266 m0 038000 D031 me Serene Situation 4 - A pipe network consists of pipeline 1 from A to B, then at B it is ‘connected to pipelines 2 and 3, where it merges again at Joint C to form a F Zup to point D. Pipelines 1, 2 and 4 are in series connection ines 2and 3 are to each other. If the rate of flow from ters/sec and assuming f= 0.02 for all pipes, compute the Tengih (a)__| Diameter Grim) 3,007 7200 3 2200 300 Situation 7 ~ The following data was oblained from laboratory 3.200 200 Cohesive specimen: mouture. content, was 225%, G, = to 3500 0 determine the approximate unit. having a mass of 224.0 g Was placed in 800 an? container with 982 cmt of water requred to fil BS 10, Rate of flow in pipe 3 in liters/ sec Gel oe tat 19, What is the nearest value tothe Total Unit Weight of the soll sample in B 231 D.378 kN/aa? 11. Rate of flow in pipe 2 in liters/ sec. poe eee i fee B. 2158 D197 5 a3 D. 658 20. What i the nearest value to the Dry Unit Weight of the soil sample in 12. Total head loss from A to D. ante ‘4. 168m © 212m oe ee 8 15m m : ote 21, Whats the void ratio o? Situation 5 A tank of carbon tetrachloride (CCI) has a mass of 500 kg and a A 0678 © 0866 volume of 0315 m3, 8 0582 D, 0784 13, Whats the weight of carbon tetrachloride in KN? peed een Situation 6 ~The ground water level ina thick, very fine sand deposits located 4 660 et 20 mbelow the ground surface, Above the fre ground waterline, the sand Se es cs is saturated by capillary action. The unt weight othe saturated sind i 203, LDS ©. 1981 kN/me. ae woes 22. What is the total stress in KPa on a horizontal plane A located 4.5 m below 415, Whati the unit weightin KN/m°? ‘he ground surface? 41655 © 1528 A762 C 36 ‘B. 15357 D. 14.82 Layee 23, Whatis the pore water pressure in kPa at this plane? A333 Cw. Situation 6 ~The table below shows the laboratory results of the seve ana ‘of a sample. Plot the grain size curve of the soil in the attached Graph T 16. Determine the nearest value to the effect B 2453 Di, 41.82 24, What is tho effective vertical tress in kPa in plane A? ‘A 0d5 C. 060 A 7212 ©. 66.83 B. 030 D. 015 B 5463 D. 87396, 17, Determine the nearest value to the coefficient of uniformity, Cy : a1 C10 Situation 9 ~ A dense silt layer has the following propertios: void ratio = 0140, BO Ds ‘effective diameter dip = 10 um, capillary constant C= 0.20 em?, Free ground 18, Classify the soil according to the Unified Classification System. Use the water level s 80 m below the ground surface. attached sheet UCS 11-2. Hydraulics ang 254 May 2003 Geotechnical Engineering Bnd te hightof cpl ie thet Caplan ries ven ah = C7 (= neh cam 3 2m Dizm 26. Find the vertical effective stress in kPa at 5 m depth. Assume unit weight of the soil above the capillary action rise and ated at 50%, B. 234 D. 134 27, Bind the vertical effective stress at 10 m depth. Assume unit weight of solids = 26.5 KN/m and that the soil above the capillary action rise and ground surface is partially saturated at 50%. ‘A 106 C194 Situation 10 - The following data were obtained from the Atterberg Limits test for a soil: Liguid Limit = 41.0 % Plastic Limit = 211 % ‘What is the plasticity index of the soil? A 215% c. 19.90% B. 25.52% D. 058% If the in situ moisture content of the soil is 30%, what is the liquidity index: of the soil? A094 © 045 B. 060 Di. -084 ‘What would be the nature of the soil? A. dense © liquid B. brittle solid D. plastic Situation 11 - A footing 6 m square carries a total load, including its own weight, ‘of 10,000 KN. The base of the footing is ata depth of 3 m below the ground surface The sol strata atthe site consist of layer Ik Footings: v= 13 oN. +y DrNy + O40 Ny, . Determine the gross foundation pressure in kPa. ‘A. 162 ©. 455 B. 393 D. 278 Determine the net foundation pressure. ‘A. 221 C489 B 357 D. 183 Calculate the factor of safety of the foundation against complete shear failure under the undrained condition (both gross and net). Side cohesion © FS yous 63 FS = 7.5 D. FSji0u=7.8 F5qa = 98 a Percent Passing, % see as 8 255 ————eeeesSsSsSSSSSS Situation 12 ~ Consider the stratified soil deposit shown in Figure O02 hee Week flowin etc layer edd na the tag Nae ey eae ea ele een coefficients of permeability of the individual iters She Rosen dlrection. Consider unit with, wees Tae Derive the expression for the equivalent coefficient of permeability in the thick each, and kin = 2x 109, kip =1 x08, fem/sec, determine the equivalent coefficient of permeability in the horizontal direction, ©. 7257 «104 D. 5061 x 108 If|=0.70, determine the total flow g in em/sec. C0674 D. 0758 Graph 11-1: Grain Size Curve 1 Particle Diameter, mm tence Lie ees ee Loe Aon zsttis| gy some al oy j a i Mydraulics and Geotechnical Engineering 258 May 2003 Solutions to May 2003 Examination Situation (L403) ee In Water: = a Draft = Swood h Swater h-o.18 b= 028 = Saeed Sword t= = 0.18 > Eq. (1) In-water In another liquid Droft= Swot j Stiguia = Sooo Seema noi Swoosh =09h =0128 > Eq. @) [Swo0a It Swooa t= 0:18 = 09h ~ 0126 > height of the block = 054m Substitute to Eq. (1): Swowa( 0.54) = 0.54 ~ 0.18 Syed = 0.667 > Specific gravity of wood k= toed Voc. p= 20238 m/s Civil Engineering Reference Vol. 2 259 ee ee aren Pat ov 2.02387 Specific energy,H= 2 +d = : a Bigrieg hye EB” Speci energy, H=11409 m Part: Slope ian 1 20238 = —1_ 3 gua Tora ©8868)" Su $0,004 3 Slope} - Part I: Shearing stress: S,=7RS = 9.81(0.8868)(0,00094) 5, = 0.00818 kPa = 8.18 Pa 2 Situation 3(7 10 9) Poyha =981(1)(2%1) F=1962kN y= 42) = 0.667 m eM =0) FxyaTx25 19.62(08567) = 257 T=5220 kN From the FBD of the concrete block ReFy=0) T+BE-W BE =1Veon ™ 981 Vere We tone Vane * 23.6 Vane 5.2324 981 Vene= 23.6 Vere Vesne= 0.3796 m9? Hydraulics and Geotechnical Engineering 260 May 2003 DSiraation #00101) @ Qi=Qu=10L/s i= Qu=0.01 m'/s ty SPS 0,0826(0.02)(3000)(0.01)? 72) 02 eee 0.0826(0.02)(2200)(Q2)* 035 110826(0.02)3200\05)*. _ 16.509 ge 02° = £10826(0.02)(2800)(0.01)* = (iene 100s ae =1.549 m sae ee = 1495.64 08 ae ig = 00452m es Pa ea oa= 15520 08 Qn= 3323 Os > Eq. (1) [or + Q= 001] 3.323 03+ Qs=0.01 Qs= 0.00251 m/s Q=251 Ys Substitute Qsto Fa.) (Qr= 33230000231) = 0.007687 m3/s = 74687 Ls FHL Fat he he) Fre San aos (00076872 + 00152 HL=1683 Civil Engineering Reference Vol. 2 ee, © Situation 5 (13 10 15) Mass, MI = 500 keg, Volume, V= 0.315 m? Weight Weight, W= Mg = 500(9.81) Weight, W 4,905 kN = 4.905 kN Mass density = 15873081) = 15571 N/m Y= 1857 kNj/mnt © Situation 6 (16 to 18 i i he dlametr ofthe particles 2f which is isan important value regulating, lor through significantly influence the mechanical behavior of soils. For this problem, Dip= 0.149 mm ‘The Average Grain Size diameter ofthe sil is Dee. Mydrantics and 262, May 2003 Geotechnical Engineering Part 2: Coefficient of uniformity: Percent Passing, % Sages oes 28 oo 8 Particle Diameter, mm Deo G= Se Dy Dy=0349) From the grain size curve shown, Dig 22 mm 22 = 22 ats ES 0149 Civil Engineering Reference Vol. 2 UNIFIED CLASSIFICATION SYSTEM (USCSa1-2) rene [ae esr cre CHD 6 Ce-DAREy Oat Bement 43 Tsay eco tear See, vn et ce ese SE usin) sts me ‘ba sean aT Raa Part 3Clasifation of soi ce On)? graphy Dor = 068 mm oo a5 estou Se Sree = 121 (Betwoon 1&3)

Você também pode gostar